Programs & Examples On #Controlling

VBoxManage: error: Failed to create the host-only adapter

For Mac OS X 10.9.3 and Vagrant 1.6.3 and VirtualBox 4.3.6 this problem was fixed with restarting the VirtualBox

sudo "/Library/Application Support/VirtualBox/LaunchDaemons/VirtualBoxStartup.sh" restart

How to create a sleep/delay in nodejs that is Blocking?

I found something almost working here https://stackoverflow.com/questions/21819858/how-to-wrap-async-function-calls-into-a-sync-function-in-node-js-or-ja vascript

`function AnticipatedSyncFunction(){
    var ret;
    setTimeout(function(){
        var startdate = new Date()
        ret = "hello" + startdate;
    },3000);
    while(ret === undefined) {
       require('deasync').runLoopOnce();
    }
    return ret;    
}


var output = AnticipatedSyncFunction();
var startdate = new Date()
console.log(startdate)
console.log("output="+output);`

The unique problem is the date printed isn't correct but the process at least is sequential.

Controlling execution order of unit tests in Visual Studio

I dont see anyone mentioning the ClassInitialize attribute method. The attributes are pretty straight forward.

Create methods that are marked with either the [ClassInitialize()] or [TestInitialize()] attribute to prepare aspects of the environment in which your unit test will run. The purpose of this is to establish a known state for running your unit test. For example, you may use the [ClassInitialize()] or the [TestInitialize()] method to copy, alter, or create certain data files that your test will use.

Create methods that are marked with either the [ClassCleanup()] or [TestCleanUp{}] attribute to return the environment to a known state after a test has run. This might mean the deletion of files in folders or the return of a database to a known state. An example of this is to reset an inventory database to an initial state after testing a method that is used in an order-entry application.

  • [ClassInitialize()] Use ClassInitialize to run code before you run the first test in the class.

  • [ClassCleanUp()] Use ClassCleanup to run code after all tests in a class have run.

  • [TestInitialize()] Use TestInitialize to run code before you run each test.

  • [TestCleanUp()] Use TestCleanup to run code after each test has run.

Controlling fps with requestAnimationFrame?

I suggest wrapping your call to requestAnimationFrame in a setTimeout:

const fps = 25;
function animate() {
  // perform some animation task here

  setTimeout(() => {
    requestAnimationFrame(animate);
  }, 1000 / fps);
}
animate();

You need to call requestAnimationFrame from within setTimeout, rather than the other way around, because requestAnimationFrame schedules your function to run right before the next repaint, and if you delay your update further using setTimeout you will have missed that time window. However, doing the reverse is sound, since you’re simply waiting a period of time before making the request.

How to achieve pagination/table layout with Angular.js?

Here is my solution. @Maxim Shoustin's solution has some issue with sorting. I also wrap the whole thing to a directive. The only dependency is UI.Bootstrap.pagination, which did a great job on pagination.

Here is the plunker

Here is the github source code.

Playing HTML5 video on fullscreen in android webview

Edit: please see my other answer, as you probably don't need this now.

As you said, in API levels 11+ a HTML5VideoFullScreen$VideoSurfaceView is passed. But I don't think you are right when you say that "it doens't have a MediaPlayer".

This is the way to reach the MediaPlayer instance from the HTML5VideoFullScreen$VideoSurfaceView instance using reflection:

@SuppressWarnings("rawtypes")
Class c1 = Class.forName("android.webkit.HTML5VideoFullScreen$VideoSurfaceView");
Field f1 = c1.getDeclaredField("this$0");
f1.setAccessible(true);

@SuppressWarnings("rawtypes")
Class c2 = f1.getType().getSuperclass();
Field f2 = c2.getDeclaredField("mPlayer");
f2.setAccessible(true);

Object ___html5VideoViewInstance = f1.get(focusedChild); // Look at the code in my other answer to this same question to see whats focusedChild

Object ___mpInstance = f2.get(___html5VideoViewInstance); // This is the MediaPlayer instance.

So, now you could set the onCompletion listener of the MediaPlayer instance like this:

OnCompletionListener ocl = new OnCompletionListener()
{
    @Override
    public void onCompletion(MediaPlayer mp)
    {
        // Do stuff
    }
};

Method m1 = f2.getType().getMethod("setOnCompletionListener", new Class[] { Class.forName("android.media.MediaPlayer$OnCompletionListener") });
m1.invoke(___mpInstance, ocl);

The code doesn't fail but I'm not completely sure if that onCompletion listener will really be called or if it could be useful to your situation. But just in case someone would like to try it.

Turning off some legends in a ggplot

You can use guide=FALSE in scale_..._...() to suppress legend.

For your example you should use scale_colour_continuous() because length is continuous variable (not discrete).

(p3 <- ggplot(mov, aes(year, rating, colour = length, shape = mpaa)) +
   scale_colour_continuous(guide = FALSE) +
   geom_point()
)

Or using function guides() you should set FALSE for that element/aesthetic that you don't want to appear as legend, for example, fill, shape, colour.

p0 <- ggplot(mov, aes(year, rating, colour = length, shape = mpaa)) +
  geom_point()    
p0+guides(colour=FALSE)

UPDATE

Both provided solutions work in new ggplot2 version 2.0.0 but movies dataset is no longer present in this library. Instead you have to use new package ggplot2movies to check those solutions.

library(ggplot2movies)
data(movies)
mov <- subset(movies, length != "")

Can not deserialize instance of java.util.ArrayList out of VALUE_STRING

For people that find this question by searching for the error message, you can also see this error if you make a mistake in your @JsonProperty annotations such that you annotate a List-typed property with the name of a single-valued field:

@JsonProperty("someSingleValuedField") // Oops, should have been "someMultiValuedField"
public List<String> getMyField() { // deserialization fails - single value into List
  return myField;
}

Live-stream video from one android phone to another over WiFi

If you do not need the recording and playback functionality in your app, using off-the-shelf streaming app and player is a reasonable choice.

If you do need them to be in your app, however, you will have to look into MediaRecorder API (for the server/camera app) and MediaPlayer (for client/player app).

Quick sample code for the server:

// this is your network socket
ParcelFileDescriptor pfd = ParcelFileDescriptor.fromSocket(socket);
mCamera = getCameraInstance();
mMediaRecorder = new MediaRecorder();
mCamera.unlock();
mMediaRecorder.setCamera(mCamera);
mMediaRecorder.setAudioSource(MediaRecorder.AudioSource.CAMCORDER);
mMediaRecorder.setVideoSource(MediaRecorder.VideoSource.CAMERA);
// this is the unofficially supported MPEG2TS format, suitable for streaming (Android 3.0+)
mMediaRecorder.setOutputFormat(8);
mMediaRecorder.setAudioEncoder(MediaRecorder.AudioEncoder.DEFAULT);
mMediaRecorder.setVideoEncoder(MediaRecorder.VideoEncoder.DEFAULT);
mediaRecorder.setOutputFile(pfd.getFileDescriptor());
mMediaRecorder.setPreviewDisplay(mPreview.getHolder().getSurface());
mMediaRecorder.prepare();
mMediaRecorder.start();

On the player side it is a bit tricky, you could try this:

// this is your network socket, connected to the server
ParcelFileDescriptor pfd = ParcelFileDescriptor.fromSocket(socket);
mMediaPlayer = new MediaPlayer();
mMediaPlayer.setDataSource(pfd.getFileDescriptor());
mMediaPlayer.prepare();
mMediaPlayer.start();

Unfortunately mediaplayer tends to not like this, so you have a couple of options: either (a) save data from socket to file and (after you have a bit of data) play with mediaplayer from file, or (b) make a tiny http proxy that runs locally and can accept mediaplayer's GET request, reply with HTTP headers, and then copy data from the remote server to it. For (a) you would create the mediaplayer with a file path or file url, for (b) give it a http url pointing to your proxy.

See also:

Stream live video from phone to phone using socket fd

MediaPlayer stutters at start of mp3 playback

Controlling Spacing Between Table Cells

Check this fiddle. You are going to need to take a look at using border-collapse and border-spacing. There are some quirks for IE (as usual). This is based on an answer to this question.

_x000D_
_x000D_
table.test td {
  background-color: lime;
  margin: 12px 12px 12px 12px;
  padding: 12px 12px 12px 12px;
}

table.test {
  border-collapse: separate;
  border-spacing: 10px;
  *border-collapse: expression('separate', cellSpacing='10px');
}
_x000D_
<table class="test">
  <tr>
    <td>Cell</td>
    <td>Cell</td>
    <td>Cell</td>
  </tr>
  <tr>
    <td>Cell</td>
    <td>Cell</td>
    <td>Cell</td>
  </tr>
  <tr>
    <td>Cell</td>
    <td>Cell</td>
    <td>Cell</td>
  </tr>
</table>
_x000D_
_x000D_
_x000D_

Update Query with INNER JOIN between tables in 2 different databases on 1 server

It is explained here http://erabhinavrana.blogspot.in/2014/01/how-to-execute-update-query-by-applying.html

It also has other useful code snippets which are commonly used.

update <dbname of 1st table>.<table name of 1st table> A INNER JOIN <dbname of 2nd table>.<table name of 2nd table> RA ON A.<field name of table 1>=RA.<field name of table 2> SET A.<field name of table 1 to be updated>=RA.<field name of table 2 to set value in table 1>

Replace data in <> with your appropriate values.

That's It. source:

http://www.dynamic-coders.com/how-to-update-two-different-tables-in-different-databases-on-same-server

Inversion of Control vs Dependency Injection

//ICO , DI ,10 years back , this was they way:

public class  AuditDAOImpl implements Audit{

    //dependency
    AuditDAO auditDAO = null;
        //Control of the AuditDAO is with AuditDAOImpl because its creating the object
    public AuditDAOImpl () {
        this.auditDAO = new AuditDAO ();
    }
}

Now with Spring 3,4 or latest its like below

public class  AuditDAOImpl implements Audit{

    //dependency

     //Now control is shifted to Spring. Container find the object and provide it. 
    @Autowired
    AuditDAO auditDAO = null;

}

Overall the control is inverted from old concept of coupled code to the frameworks like Spring which makes the object available. So that's IOC as far as I know and Dependency injection as you know when we inject the dependent object into another object using Constructor or setters . Inject basically means passing it as an argument. In spring we have XML & annotation based configuration where we define bean object and pass the dependent object with Constructor or setter injection style.

Controlling a USB power supply (on/off) with Linux

echo '2-1' |sudo tee /sys/bus/usb/drivers/usb/unbind

works for ubuntu

Controlling Maven final name of jar artifact

I am using the following

        ....
        <plugin>
            <groupId>org.apache.maven.plugins</groupId>
            <artifactId>maven-jar-plugin</artifactId>
            <version>3.0.2</version>
            <configuration>
                <finalName>${project.groupId}/${project.artifactId}-${baseVersion}.${monthlyVersion}.${instanceVersion}</finalName>
            </configuration>
        </plugin>
        ....

This way you can define each value individually or pragmatically from Jenkins of some other system.

mvn package -DbaseVersion=1 -monthlyVersion=2 -instanceVersion=3

This will place a folder target\{group.id}\projectName-1.2.3.jar

A better way to save time might be

        ....
        <plugin>
            <groupId>org.apache.maven.plugins</groupId>
            <artifactId>maven-jar-plugin</artifactId>
            <version>3.0.2</version>
            <configuration>
                <finalName>${project.groupId}/${project.artifactId}-${baseVersion}</finalName>
            </configuration>
        </plugin>
        ....

Like the same except I use on variable.

  mvn package -DbaseVersion=0.3.4

This will place a folder target\{group.id}\projectName-1.2.3.jar

you can also use outputDirectory inside of configuration to specify a location you may want the package to be located.

Controlling number of decimal digits in print output in R

If you are producing the entire output yourself, you can use sprintf(), e.g.

> sprintf("%.10f",0.25)
[1] "0.2500000000"

specifies that you want to format a floating point number with ten decimal points (in %.10f the f is for float and the .10 specifies ten decimal points).

I don't know of any way of forcing R's higher level functions to print an exact number of digits.

Displaying 100 digits does not make sense if you are printing R's usual numbers, since the best accuracy you can get using 64-bit doubles is around 16 decimal digits (look at .Machine$double.eps on your system). The remaining digits will just be junk.

Controlling mouse with Python

Tested on WinXP, Python 2.6 (3.x also tested) after installing pywin32 (pywin32-214.win32-py2.6.exe in my case):

import win32api, win32con
def click(x,y):
    win32api.SetCursorPos((x,y))
    win32api.mouse_event(win32con.MOUSEEVENTF_LEFTDOWN,x,y,0,0)
    win32api.mouse_event(win32con.MOUSEEVENTF_LEFTUP,x,y,0,0)
click(10,10)

Globally catch exceptions in a WPF application?

In addition what others mentioned here, note that combining the Application.DispatcherUnhandledException (and its similars) with

<configuration>
  <runtime>  
    <legacyUnhandledExceptionPolicy enabled="1" />
  </runtime>
</configuration>

in the app.config will prevent your secondary threads exception from shutting down the application.

Javascript: Easier way to format numbers?

Just finished up a js library for formatting numbers Numeral.js. It handles decimals, dollars, percentages and even time formatting.

jQuery DataTables: control table width

"fnInitComplete": function() {
    $("#datatables4_wrapper").css("width","60%");
 }

This worked fine to adjust the whole table width. Thanks @Peter Drinnan!

Best way to access a control on another form in Windows Forms?

This looks like a prime candidate for separating the presentation from the data model. In this case, your preferences should be stored in a separate class that fires event updates whenever a particular property changes (look into INotifyPropertyChanged if your properties are a discrete set, or into a single event if they are more free-form text-based keys).

In your tree view, you'll make the changes to your preferences model, it will then fire an event. In your other forms, you'll subscribe to the changes that you're interested in. In the event handler you use to subscribe to the property changes, you use this.InvokeRequired to see if you are on the right thread to make the UI call, if not, then use this.BeginInvoke to call the desired method to update the form.

What is the easiest way to parse an INI file in Java?

It is just as simple as this.....

//import java.io.FileInputStream;
//import java.io.FileInputStream;

Properties prop = new Properties();
//c:\\myapp\\config.ini is the location of the ini file
//ini file should look like host=localhost
prop.load(new FileInputStream("c:\\myapp\\config.ini"));
String host = prop.getProperty("host");

How to reset sequence in postgres and fill id column with new data?

Inspired by the other answers here, I created an SQL function to do a sequence migration. The function moves a primary key sequence to a new contiguous sequence starting with any value (>= 1) either inside or outside the existing sequence range.

I explain here how I used this function in a migration of two databases with the same schema but different values into one database.

First, the function (which prints the generated SQL commands so that it is clear what is actually happening):

CREATE OR REPLACE FUNCTION migrate_pkey_sequence
  ( arg_table      text
  , arg_column     text
  , arg_sequence   text
  , arg_next_value bigint  -- Must be >= 1
  )
RETURNS int AS $$
DECLARE
  result int;
  curr_value bigint = arg_next_value - 1;
  update_column1 text := format
    ( 'UPDATE %I SET %I = nextval(%L) + %s'
    , arg_table
    , arg_column
    , arg_sequence
    , curr_value
    );
  alter_sequence text := format
    ( 'ALTER SEQUENCE %I RESTART WITH %s'
    , arg_sequence
    , arg_next_value
    );
  update_column2 text := format
    ( 'UPDATE %I SET %I = DEFAULT'
    , arg_table
    , arg_column
    );
  select_max_column text := format
    ( 'SELECT coalesce(max(%I), %s) + 1 AS nextval FROM %I'
    , arg_column
    , curr_value
    , arg_table
    );
BEGIN
  -- Print the SQL command before executing it.
  RAISE INFO '%', update_column1;
  EXECUTE update_column1;
  RAISE INFO '%', alter_sequence;
  EXECUTE alter_sequence;
  RAISE INFO '%', update_column2;
  EXECUTE update_column2;
  EXECUTE select_max_column INTO result;
  RETURN result;
END $$ LANGUAGE plpgsql;

The function migrate_pkey_sequence takes the following arguments:

  1. arg_table: table name (e.g. 'example')
  2. arg_column: primary key column name (e.g. 'id')
  3. arg_sequence: sequence name (e.g. 'example_id_seq')
  4. arg_next_value: next value for the column after migration

It performs the following operations:

  1. Move the primary key values to a free range. I assume that nextval('example_id_seq') follows max(id) and that the sequence starts with 1. This also handles the case where arg_next_value > max(id).
  2. Move the primary key values to the contiguous range starting with arg_next_value. The order of key values are preserved but holes in the range are not preserved.
  3. Print the next value that would follow in the sequence. This is useful if you want to migrate the columns of another table and merge with this one.

To demonstrate, we use a sequence and table defined as follows (e.g. using psql):

# CREATE SEQUENCE example_id_seq
  START WITH 1
  INCREMENT BY 1
  NO MINVALUE
  NO MAXVALUE
  CACHE 1;
# CREATE TABLE example
  ( id bigint NOT NULL DEFAULT nextval('example_id_seq'::regclass)
  );

Then, we insert some values (starting, for example, at 3):

# ALTER SEQUENCE example_id_seq RESTART WITH 3;
# INSERT INTO example VALUES (DEFAULT), (DEFAULT), (DEFAULT);
-- id: 3, 4, 5

Finally, we migrate the example.id values to start with 1.

# SELECT migrate_pkey_sequence('example', 'id', 'example_id_seq', 1);
INFO:  00000: UPDATE example SET id = nextval('example_id_seq') + 0
INFO:  00000: ALTER SEQUENCE example_id_seq RESTART WITH 1
INFO:  00000: UPDATE example SET id = DEFAULT
 migrate_pkey_sequence
-----------------------
                     4
(1 row)

The result:

# SELECT * FROM example;
 id
----
  1
  2
  3
(3 rows)

target="_blank" vs. target="_new"

  • _blank as a target value will spawn a new window every time,
  • _new will only spawn one new window.

Also, every link clicked with a target value of _new will replace the page loaded in the previously spawned window.

You can click here When to use _blank or _new to try it out for yourself.

How do I convert from a money datatype in SQL server?

This looks like a formating issue to me.
As far as SQL Server's money type is concerned 0 == 0.00

If you're trying to display 0 in say c# rather then 0.00 you should convert it to a string, and format it as you want. (or truncate it.)

How can I create a copy of an object in Python?

Shallow copy with copy.copy()

#!/usr/bin/env python3

import copy

class C():
    def __init__(self):
        self.x = [1]
        self.y = [2]

# It copies.
c = C()
d = copy.copy(c)
d.x = [3]
assert c.x == [1]
assert d.x == [3]

# It's shallow.
c = C()
d = copy.copy(c)
d.x[0] = 3
assert c.x == [3]
assert d.x == [3]

Deep copy with copy.deepcopy()

#!/usr/bin/env python3
import copy
class C():
    def __init__(self):
        self.x = [1]
        self.y = [2]
c = C()
d = copy.deepcopy(c)
d.x[0] = 3
assert c.x == [1]
assert d.x == [3]

Documentation: https://docs.python.org/3/library/copy.html

Tested on Python 3.6.5.

Mobile overflow:scroll and overflow-scrolling: touch // prevent viewport "bounce"

you could try

$('*').not('#div').bind('touchmove', false);

add this if necessary

$('#div').bind('touchmove');

note that everything is fixed except #div

Status bar and navigation bar appear over my view's bounds in iOS 7

The simplest trick is to open the NIB file and do these two simple steps:

  1. Just toggle that and set it to the one you prefer:

Enter image description here

  1. Select those UIView's/UIIMageView's/... that you want to be moved down. In my case only the logo was overlapped an I've set the delta to +15; (OR -15 if you chose iOS 7 in step 1)

Enter image description here

And the result:

Before After

How to add headers to OkHttp request interceptor?

There is yet an another way to add interceptors in your OkHttp3 (latest version as of now) , that is you add the interceptors to your Okhttp builder

okhttpBuilder.networkInterceptors().add(chain -> {
 //todo add headers etc to your AuthorisedRequest

  return chain.proceed(yourAuthorisedRequest);
});

and finally build your okHttpClient from this builder

OkHttpClient client = builder.build();

AngularJS: Insert HTML from a string

you can also use $sce.trustAsHtml('"<h1>" + str + "</h1>"'),if you want to know more detail, please refer to $sce

How to save a figure in MATLAB from the command line?

imwrite(A,filename) writes image data A to the file specified by filename, inferring the file format from the extension

Convert 4 bytes to int

You should put it into a function like this:

public static int toInt(byte[] bytes, int offset) {
  int ret = 0;
  for (int i=0; i<4 && i+offset<bytes.length; i++) {
    ret <<= 8;
    ret |= (int)bytes[i] & 0xFF;
  }
  return ret;
}

Example:

byte[] bytes = new byte[]{-2, -4, -8, -16};
System.out.println(Integer.toBinaryString(toInt(bytes, 0)));

Output:

11111110111111001111100011110000

This takes care of running out of bytes and correctly handling negative byte values.

I'm unaware of a standard function for doing this.

Issues to consider:

  1. Endianness: different CPU architectures put the bytes that make up an int in different orders. Depending on how you come up with the byte array to begin with you may have to worry about this; and

  2. Buffering: if you grab 1024 bytes at a time and start a sequence at element 1022 you will hit the end of the buffer before you get 4 bytes. It's probably better to use some form of buffered input stream that does the buffered automatically so you can just use readByte() repeatedly and not worry about it otherwise;

  3. Trailing Buffer: the end of the input may be an uneven number of bytes (not a multiple of 4 specifically) depending on the source. But if you create the input to begin with and being a multiple of 4 is "guaranteed" (or at least a precondition) you may not need to concern yourself with it.

to further elaborate on the point of buffering, consider the BufferedInputStream:

InputStream in = new BufferedInputStream(new FileInputStream(file), 1024);

Now you have an InputStream that automatically buffers 1024 bytes at a time, which is a lot less awkward to deal with. This way you can happily read 4 bytes at a time and not worry about too much I/O.

Secondly you can also use DataInputStream:

InputStream in = new DataInputStream(new BufferedInputStream(
                     new FileInputStream(file), 1024));
byte b = in.readByte();

or even:

int i = in.readInt();

and not worry about constructing ints at all.

How do I disable orientation change on Android?

Update April 2013: Don't do this. It wasn't a good idea in 2009 when I first answered the question and it really isn't a good idea now. See this answer by hackbod for reasons:

Avoid reloading activity with asynctask on orientation change in android

Add android:configChanges="keyboardHidden|orientation" to your AndroidManifest.xml. This tells the system what configuration changes you are going to handle yourself - in this case by doing nothing.

<activity android:name="MainActivity"
     android:screenOrientation="portrait"
     android:configChanges="keyboardHidden|orientation">

See Developer reference configChanges for more details.

However, your application can be interrupted at any time, e.g. by a phone call, so you really should add code to save the state of your application when it is paused.

Update: As of Android 3.2, you also need to add "screenSize":

<activity
    android:name="MainActivity"
    android:screenOrientation="portrait"
    android:configChanges="keyboardHidden|orientation|screenSize">

From Developer guide Handling the Configuration Change Yourself

Caution: Beginning with Android 3.2 (API level 13), the "screen size" also changes when the device switches between portrait and landscape orientation. Thus, if you want to prevent runtime restarts due to orientation change when developing for API level 13 or higher (as declared by the minSdkVersion and targetSdkVersion attributes), you must include the "screenSize" value in addition to the "orientation" value. That is, you must declare android:configChanges="orientation|screenSize". However, if your application targets API level 12 or lower, then your activity always handles this configuration change itself (this configuration change does not restart your activity, even when running on an Android 3.2 or higher device).

add created_at and updated_at fields to mongoose schemas

UPDATE: (5 years later)

Note: If you decide to use Kappa Architecture (Event Sourcing + CQRS), then you do not need updated date at all. Since your data is an immutable, append-only event log, you only ever need event created date. Similar to the Lambda Architecture, described below. Then your application state is a projection of the event log (derived data). If you receive a subsequent event about existing entity, then you'll use that event's created date as updated date for your entity. This is a commonly used (and commonly misunderstood) practice in miceroservice systems.

UPDATE: (4 years later)

If you use ObjectId as your _id field (which is usually the case), then all you need to do is:

let document = {
  updatedAt: new Date(),
}

Check my original answer below on how to get the created timestamp from the _id field. If you need to use IDs from external system, then check Roman Rhrn Nesterov's answer.

UPDATE: (2.5 years later)

You can now use the #timestamps option with mongoose version >= 4.0.

let ItemSchema = new Schema({
  name: { type: String, required: true, trim: true }
},
{
  timestamps: true
});

If set timestamps, mongoose assigns createdAt and updatedAt fields to your schema, the type assigned is Date.

You can also specify the timestamp fileds' names:

timestamps: { createdAt: 'created_at', updatedAt: 'updated_at' }

Note: If you are working on a big application with critical data you should reconsider updating your documents. I would advise you to work with immutable, append-only data (lambda architecture). What this means is that you only ever allow inserts. Updates and deletes should not be allowed! If you would like to "delete" a record, you could easily insert a new version of the document with some timestamp/version filed and then set a deleted field to true. Similarly if you want to update a document – you create a new one with the appropriate fields updated and the rest of the fields copied over.Then in order to query this document you would get the one with the newest timestamp or the highest version which is not "deleted" (the deleted field is undefined or false`).

Data immutability ensures that your data is debuggable – you can trace the history of every document. You can also rollback to previous version of a document if something goes wrong. If you go with such an architecture ObjectId.getTimestamp() is all you need, and it is not Mongoose dependent.


ORIGINAL ANSWER:

If you are using ObjectId as your identity field you don't need created_at field. ObjectIds have a method called getTimestamp().

ObjectId("507c7f79bcf86cd7994f6c0e").getTimestamp()

This will return the following output:

ISODate("2012-10-15T21:26:17Z")

More info here How do I extract the created date out of a Mongo ObjectID

In order to add updated_at filed you need to use this:

var ArticleSchema = new Schema({
  updated_at: { type: Date }
  // rest of the fields go here
});

ArticleSchema.pre('save', function(next) {
  this.updated_at = Date.now();
  next();
});

Having services in React application

Same situation: Having done multiple Angular projects and moving to React, not having a simple way to provide services through DI seems like a missing piece (the particulars of the service aside).

Using context and ES7 decorators we can come close:

https://jaysoo.ca/2015/06/09/react-contexts-and-dependency-injection/

Seems these guys have taken it a step further / in a different direction:

http://blog.wolksoftware.com/dependency-injection-in-react-powered-inversifyjs

Still feels like working against the grain. Will revisit this answer in 6 months time after undertaking a major React project.

EDIT: Back 6 months later with some more React experience. Consider the nature of the logic:

  1. Is it tied (only) to UI? Move it into a component (accepted answer).
  2. Is it tied (only) to state management? Move it into a thunk.
  3. Tied to both? Move to separate file, consume in component through a selector and in thunks.

Some also reach for HOCs for reuse but for me the above covers almost all use cases. Also, consider scaling state management using ducks to keep concerns separate and state UI-centric.

Prevent the keyboard from displaying on activity start

Function to hide the keyboard.

public static void hideKeyboard(Activity activity) {
    View view = activity.getCurrentFocus();

    if (view != null) {
        InputMethodManager inputManager = (InputMethodManager) activity.getSystemService(Context.INPUT_METHOD_SERVICE);

        inputManager.hideSoftInputFromWindow(view.getWindowToken(), InputMethodManager.HIDE_NOT_ALWAYS);
    }
}

Hide keyboard in AndroidManifext.xml file.

<activity
    android:name=".MainActivity"
    android:label="@string/app_name"
    android:theme="@style/AppTheme"
    android:windowSoftInputMode="stateHidden">

Will using 'var' affect performance?

There is no runtime performance cost to using var. Though, I would suspect there to be a compiling performance cost as the compiler needs to infer the type, though this will most likely be negligable.

Programmatically trigger "select file" dialog box

<div id="uploadButton">UPLOAD</div>
<form action="[FILE_HANDLER_URL]" style="display:none">
     <input id="myInput" type="file" />
</form>
<script>
  const uploadButton = document.getElementById('uploadButton');
  const myInput = document.getElementById('myInput');

  uploadButton.addEventListener('click', () => {
    myInput.click();
  });
</script>

Is it possible to play music during calls so that the partner can hear it ? Android

Yes it is possible in Sony ericssion xylophone w20 .I have got this phone in 2010.but yet this type of phone I have not seen.

Correct way to focus an element in Selenium WebDriver using Java

You can use JS as below:

WebDriver driver = new FirefoxDriver();
JavascriptExecutor jse = (JavascriptExecutor) driver;
jse.executeScript("document.getElementById('elementid').focus();");

SQL "IF", "BEGIN", "END", "END IF"?

Off hand the code looks right. What if you try using an 'Else' and see what happens?

IF @SchoolCategoryCode = 'Elem' 

--- We now have determined we are processing an elementary school...

BEGIN

---- Only do the following if the variable @Term equals a 3 - if it does not, skip just this first part

    IF @Term = 3
    BEGIN
        INSERT INTO @Classes

        SELECT              
            XXXXXX  
        FROM XXXX blah blah blah

        INSERT INTO @Classes    
        SELECT
        XXXXXXXX    
        FROM XXXXXX (more code) 
    END   <----(Should this be ENDIF?)
    ELSE
    BEGIN


        INSERT INTO @Classes    
        SELECT
        XXXXXXXX    
        FROM XXXXXX (more code) 
    END
END

Apache HttpClient Android (Gradle)

if you are using target sdk as 23 add below code in your build.gradle

android{
 useLibrary  'org.apache.http.legacy'
}

additional note here: dont try using the gradle versions of those files. they are broken (28.08.15). I tried over 5 hours to get it to work. it just doesnt. not working:

compile 'org.apache.httpcomponents:httpcore:4.4.1'
compile 'org.apache.httpcomponents:httpclient:4.5'

another thing dont use:

'org.apache.httpcomponents:httpclient-android:4.3.5.1'

its referring 21 api level.

Receiver not registered exception error?

For anybody who will come upon this problem and they tried all that was suggested and nothing still works, this is how I sorted my problem, instead of doing LocalBroadcastManager.getInstance(this).registerReceiver(...) I first created a local variable of type LocalBroadcastManager,

private LocalBroadcastManager lbman;

And used this variable to carry out the registering and unregistering on the broadcastreceiver, that is

lbman.registerReceiver(bReceiver);

and

lbman.unregisterReceiver(bReceiver);

How to make readonly all inputs in some div in Angular2?

If using reactive forms, you can also disable the entire form or any sub-set of controls in a FormGroup with myFormGroup.disable().

How can I access the MySQL command line with XAMPP for Windows?

To access SQL via the command line we simply navigate to the xampp folder. the folder is usually on the c:\ drive.

  1. type: cd c:\xampp\mysql\bin then press enter.
  2. type: in mysql -u root -p then press enter.
  3. in the new line enter the password of the database.
  4. type: show databases; to see all the databases that you have access to.
  5. once you know what database you want to use type (ex. cms_database), type use cms_data or any other db_name, it should say Database changed

// Bonus Note you shouldn't use the root user when working with databases. you would want to create a separate account (ex. cms_user) with special privileges to limit errors.

to do that type:

GRANT ALL PRIVILEGES IN cms_database.*
TO 'cms_user'@'localhost' 
IDENTITIED BY 'yourpassword'

double check by using the command:

SHOW GRANT FOR 'cms_user'@localhost;

if you already have the name of the user and the specific database you wish to use, change step 2 to this.

mysql -u cms_user -p cms_database 

How to get the scroll bar with CSS overflow on iOS

Apply this code in your css

::-webkit-scrollbar{

    -webkit-appearance: none;
    width: 7px;

}

::-webkit-scrollbar-thumb {

    border-radius: 4px;
    background-color: rgba(0,0,0,.5); 
    -webkit-box-shadow: 0 0 1px rgba(255,255,255,.5);
}

How can I pad a value with leading zeros?

Even later to the party.

function zfill(num, len) {
  return(0 > num ? "-" : "") + (Math.pow(10, len) <= Math.abs(num) ? "0" + Math.abs(num) : Math.pow(10, len) + Math.abs(num)).toString().substr(1)
}

This handles negatives and situations where the number is longer than the field width. And floating-point.

javascript node.js next()

It is naming convention used when passing callbacks in situations that require serial execution of actions, e.g. scan directory -> read file data -> do something with data. This is in preference to deeply nesting the callbacks. The first three sections of the following article on Tim Caswell's HowToNode blog give a good overview of this:

http://howtonode.org/control-flow

Also see the Sequential Actions section of the second part of that posting:

http://howtonode.org/control-flow-part-ii

How can I find the maximum value and its index in array in MATLAB?

In case of a 2D array (matrix), you can use:

[val, idx] = max(A, [], 2);

The idx part will contain the column number of containing the max element of each row.

Determine whether an array contains a value

You can use _.indexOf method or if you don't want to include whole Underscore.js library in your app, you can have a look how they did it and extract necessary code.

    _.indexOf = function(array, item, isSorted) {
    if (array == null) return -1;
    var i = 0, l = array.length;
    if (isSorted) {
      if (typeof isSorted == 'number') {
        i = (isSorted < 0 ? Math.max(0, l + isSorted) : isSorted);
      } else {
        i = _.sortedIndex(array, item);
        return array[i] === item ? i : -1;
      }
    }
    if (nativeIndexOf && array.indexOf === nativeIndexOf) return array.indexOf(item, isSorted);
    for (; i < l; i++) if (array[i] === item) return i;
    return -1;
  };

SQL to Query text in access with an apostrophe in it

Escape the apostrophe in O'Neal by writing O''Neal (two apostrophes).

Shortest way to check for null and assign another value if not

To extend @Dave's answer...if planRec.approved_by is already a string

this.approved_by = planRec.approved_by ?? "";

Best way to incorporate Volley (or other library) into Android Studio project

UPDATE:

compile 'com.android.volley:volley:1.0.0'

OLD ANSWER: You need the next in your build.gradle of your app module:

dependencies {
        compile 'com.mcxiaoke.volley:library:1.0.19'
        (Rest of your dependencies)

    }

This is not the official repo but is a highly trusted one.

What is the meaning of "POSIX"?

A specification (blueprint) about how to make an OS compatible with late UNIX OS (may God bless him!). This is why macOS and GNU/Linux have very similar terminal command lines, GUI's, libraries, etc. Because they both were designed according to POSIX blueprint.

POSIX does not tell engineers and programmers how to code but what to code.

Adding item to Dictionary within loop

# Let's add key:value to a dictionary, the functional way 
# Create your dictionary class 
class my_dictionary(dict): 
    # __init__ function 
    def __init__(self): 
        self = dict()   
    # Function to add key:value 
    def add(self, key, value): 
        self[key] = value 
# Main Function 
dict_obj = my_dictionary() 
limit = int(input("Enter the no of key value pair in a dictionary"))
c=0
while c < limit :   
    dict_obj.key = input("Enter the key: ") 
    dict_obj.value = input("Enter the value: ") 
    dict_obj.add(dict_obj.key, dict_obj.value) 
    c += 1
print(dict_obj) 

android:layout_height 50% of the screen size

To achieve this feat, define a outer linear layout with a weightSum={amount of weight to distribute}.

it defines the maximum weight sum. If unspecified, the sum is computed by adding the layout_weight of all of the children. This can be used for instance to give a single child 50% of the total available space by giving it a layout_weight of 0.5 and setting the weightSum to 1.0.Another example would be set weightSum=2, and if the two children set layout_weight=1 then each would get 50% of the available space.

WeightSum is dependent on the amount of children in the parent layout.

ScalaTest in sbt: is there a way to run a single test without tags?

I don't see a way to run a single untagged test within a test class but I am providing my workflow since it seems to be useful for anyone who runs into this question.

From within a sbt session:

test:testOnly *YourTestClass

(The asterisk is a wildcard, you could specify the full path com.example.specs.YourTestClass.)

All tests within that test class will be executed. Presumably you're most concerned with failing tests, so correct any failing implementations and then run:

test:testQuick

... which will only execute tests that failed. (Repeating the most recently executed test:testOnly command will be the same as test:testQuick in this case, but if you break up your test methods into appropriate test classes you can use a wildcard to make test:testQuick a more efficient way to re-run failing tests.)

Note that the nomenclature for test in ScalaTest is a test class, not a specific test method, so all untagged methods are executed.

If you have too many test methods in a test class break them up into separate classes or tag them appropriately. (This could be a signal that the class under test is in violation of single responsibility principle and could use a refactoring.)

Converting SVG to PNG using C#

I'm using Batik for this. The complete Delphi code:

procedure ExecNewProcess(ProgramName : String; Wait: Boolean);
var
  StartInfo : TStartupInfo;
  ProcInfo : TProcessInformation;
  CreateOK : Boolean;
begin
  FillChar(StartInfo, SizeOf(TStartupInfo), #0);
  FillChar(ProcInfo, SizeOf(TProcessInformation), #0);
  StartInfo.cb := SizeOf(TStartupInfo);
  CreateOK := CreateProcess(nil, PChar(ProgramName), nil, nil, False,
              CREATE_NEW_PROCESS_GROUP + NORMAL_PRIORITY_CLASS,
              nil, nil, StartInfo, ProcInfo);
  if CreateOK then begin
    //may or may not be needed. Usually wait for child processes
    if Wait then
      WaitForSingleObject(ProcInfo.hProcess, INFINITE);
  end else
    ShowMessage('Unable to run ' + ProgramName);

  CloseHandle(ProcInfo.hProcess);
  CloseHandle(ProcInfo.hThread);
end;

procedure ConvertSVGtoPNG(aFilename: String);
const
  ExecLine = 'c:\windows\system32\java.exe -jar C:\Apps\batik-1.7\batik-rasterizer.jar ';
begin
  ExecNewProcess(ExecLine + aFilename, True);
end;

How do you add a Dictionary of items into another Dictionary

Swift 3, dictionary extension:

public extension Dictionary {

    public static func +=(lhs: inout Dictionary, rhs: Dictionary) {
        for (k, v) in rhs {
            lhs[k] = v
        }
    }

}

Maven 3 Archetype for Project With Spring, Spring MVC, Hibernate, JPA

A great Spring MVC quickstart archetype is available on GitHub, courtesy of kolorobot. Good instructions are provided on how to install it to your local Maven repo and use it to create a new Spring MVC project. He’s even helpfully included the Tomcat 7 Maven plugin in the archetypical project so that the newly created Spring MVC can be run from the command line without having to manually deploy it to an application server.

Kolorobot’s example application includes the following:

  • No-xml Spring MVC 3.2 web application for Servlet 3.0 environment
  • Apache Tiles with configuration in place,
  • Bootstrap
  • JPA 2.0 (Hibernate/HSQLDB)
  • JUnit/Mockito
  • Spring Security 3.1

Cannot install packages using node package manager in Ubuntu

For me the fix was removing the node* packages and also the npm packages.

Then a fresh install as:

sudo apt-get install autoclean
sudo apt-get install nodejs-legacy
npm install

Strip HTML from Text JavaScript

simple 2 line jquery to strip the html.

 var content = "<p>checking the html source&nbsp;</p><p>&nbsp;
  </p><p>with&nbsp;</p><p>all</p><p>the html&nbsp;</p><p>content</p>";

 var text = $(content).text();//It gets you the plain text
 console.log(text);//check the data in your console

 cj("#text_area_id").val(text);//set your content to text area using text_area_id

How can I disable a tab inside a TabControl?

MyTabControl.SelectedTab.Enabled = false;

How to find the index of an element in an int array?

Simple:

public int getArrayIndex(int[] arr,int value) {
    for(int i=0;i<arr.length;i++)
        if(arr[i]==value) return i;
    return -1;
}

Generating sql insert into for Oracle

You can also use MyGeneration (free tool) to write your own sql generated scripts. There is a "insert into" script for SQL Server included in MyGeneration, which can be easily changed to run under Oracle.

update one table with data from another

Try following code. It is working for me....

UPDATE TableOne 
SET 
field1 =(SELECT TableTwo.field1 FROM TableTwo WHERE TableOne.id=TableTwo.id),
field2 =(SELECT TableTwo.field2 FROM TableTwo WHERE TableOne.id=TableTwo.id)
WHERE TableOne.id = (SELECT  TableTwo.id 
                             FROM   TableTwo 
                             WHERE  TableOne.id = TableTwo.id) 

how do I query sql for a latest record date for each user

I used this way to take the last record for each user that I have on my table. It was a query to get last location for salesman as per recent time detected on PDA devices.

CREATE FUNCTION dbo.UsersLocation()
RETURNS TABLE
AS
RETURN
Select GS.UserID, MAX(GS.UTCDateTime) 'LastDate'
From USERGPS GS
where year(GS.UTCDateTime) = YEAR(GETDATE()) 
Group By GS.UserID
GO
select  gs.UserID, sl.LastDate, gs.Latitude , gs.Longitude
        from USERGPS gs
        inner join USER s on gs.SalesManNo = s.SalesmanNo 
        inner join dbo.UsersLocation() sl on gs.UserID= sl.UserID and gs.UTCDateTime = sl.LastDate 
        order by LastDate desc

Chrome / Safari not filling 100% height of flex parent

Specifying a flex attribute to the container worked for me:

.container {
    flex: 0 0 auto;
}

This ensures the height is set and doesn't grow either.

How can I git stash a specific file?

I usually add to index changes I don't want to stash and then stash with --keep-index option.

git add app/controllers/cart_controller.php
git stash --keep-index
git reset

Last step is optional, but usually you want it. It removes changes from index.


Warning As noted in the comments, this puts everything into the stash, both staged and unstaged. The --keep-index just leaves the index alone after the stash is done. This can cause merge conflicts when you later pop the stash.

How do MySQL indexes work?

Let's suppose you have a book, probably a novel, a thick one with lots of things to read, hence lots of words. Now, hypothetically, you brought two dictionaries, consisting of only words that are only used, at least one time in the novel. All words in that two dictionaries are stored in typical alphabetical order. In hypothetical dictionary A, words are printed only once while in hypothetical dictionary B words are printed as many numbers of times it is printed in the novel. Remember, words are sorted alphabetically in both the dictionaries. Now you got stuck at some point while reading a novel and need to find the meaning of that word from anyone of those hypothetical dictionaries. What you will do? Surely you will jump to that word in a few steps to find its meaning, rather look for the meaning of each of the words in the novel, from starting, until you reach that bugging word.

This is how the index works in SQL. Consider Dictionary A as PRIMARY INDEX, Dictionary B as KEY/SECONDARY INDEX, and your desire to get for the meaning of the word as a QUERY/SELECT STATEMENT. The index will help to fetch the data at a very fast rate. Without an index, you will have to look for the data from the starting, unnecessarily time-consuming costly task.

For more about indexes and types, look this.

C++ class forward declaration

The problem is that tick() needs to know the definition of tile_tree_apple, but all it has is a forward declaration of it. You should separate the declarations and definitions like so:

tile_tree.h

#ifndef TILE_TREE_H
#define TILE_TREE_H
#include "tile.h"

class tile_tree : public tile
{
public:
    tile onDestroy();
    tile tick();
    void onCreate();
};

#endif

tile_tree.cpp:

tile tile_tree::onDestroy() {
    return *new tile_grass;
}

tile tile_tree::tick() {
     if (rand() % 20 == 0)
         return *new tile_tree_apple;
}

void tile_tree::onCreate() {
    health = rand() % 5 + 4;
    type = TILET_TREE;
}

Except you have a major problem: you’re allocating memory (with new), then copying the allocated object and returning the copy. This is called a memory leak, because there’s no way for your program to free the memory it uses. Not only that, but you’re copying a tile_tree into a tile, which discards the information that makes a tile_tree different from a tile; this is called slicing.

What you want is to return a pointer to a new tile, and make sure you call delete at some point to free the memory:

tile* tile_tree::tick() {
     if (rand() % 20 == 0)
         return new tile_tree_apple;
}

Even better would be to return a smart pointer that will handle the memory management for you:

#include <memory>

std::shared_ptr<tile> tile_tree::tick() {
     if (rand() % 20 == 0)
         return std::make_shared<tile_tree_apple>();
}

How to access List elements

It's simple

y = [['vegas','London'],['US','UK']]

for x in y:
    for a in x:
        print(a)

How do I capture the output into a variable from an external process in PowerShell?

If all you are trying to do is capture the output from a command, then this will work well.

I use it for changing system time, as [timezoneinfo]::local always produces the same information, even after you have made changes to the system. This is the only way I can validate and log the change in time zone:

$NewTime = (powershell.exe -command [timezoneinfo]::local)
$NewTime | Tee-Object -FilePath $strLFpath\$strLFName -Append

Meaning that I have to open a new PowerShell session to reload the system variables.

Pass a String from one Activity to another Activity in Android

In ActivityOne,

Intent intent = new Intent(ActivityOne.this, ActivityTwo.class);
intent.putExtra("data", somedata);
startActivity(intent);

In ActivityTwo,

Intent intent = getIntent();
String data = intent.getStringExtra("data");

VBA test if cell is in a range

From the Help:

Set isect = Application.Intersect(Range("rg1"), Range("rg2"))
If isect Is Nothing Then
    MsgBox "Ranges do not intersect"
Else
    isect.Select
End If

How to call a JavaScript function, declared in <head>, in the body when I want to call it

You can also put the JavaScript code in script tags, rather than a separate function. <script>//JS Code</script> This way the code will get executes on Page Load.

How to install a specific version of package using Composer?

Add double quotes to use caret operator in version number.

composer require middlewares/whoops "^0.4"

Subtract two dates in SQL and get days of the result

Syntax

DATEDIFF(expr1,expr2)

Description

DATEDIFF() returns (expr1 – expr2) expressed as a value in days from one date to the other. expr1 and expr2 are date or date-and-time expressions. Only the date parts of the values are used in the calculation.

@D Stanley

How to solve munmap_chunk(): invalid pointer error in C++

This happens when the pointer passed to free() is not valid or has been modified somehow. I don't really know the details here. The bottom line is that the pointer passed to free() must be the same as returned by malloc(), realloc() and their friends. It's not always easy to spot what the problem is for a novice in their own code or even deeper in a library. In my case, it was a simple case of an undefined (uninitialized) pointer related to branching.

The free() function frees the memory space pointed to by ptr, which must have been returned by a previous call to malloc(), calloc() or realloc(). Otherwise, or if free(ptr) has already been called before, undefined behavior occurs. If ptr is NULL, no operation is performed. GNU 2012-05-10 MALLOC(3)

char *words; // setting this to NULL would have prevented the issue

if (condition) {
    words = malloc( 512 );

    /* calling free sometime later works here */

    free(words)
} else {

    /* do not allocate words in this branch */
}

/* free(words);  -- error here --
*** glibc detected *** ./bin: munmap_chunk(): invalid pointer: 0xb________ ***/

There are many similar questions here about the related free() and rellocate() functions. Some notable answers providing more details:

*** glibc detected *** free(): invalid next size (normal): 0x0a03c978 ***
*** glibc detected *** sendip: free(): invalid next size (normal): 0x09da25e8 ***
glibc detected, realloc(): invalid pointer


IMHO running everything in a debugger (Valgrind) is not the best option because errors like this are often caused by inept or novice programmers. It's more productive to figure out the issue manually and learn how to avoid it in the future.

PHP - Session destroy after closing browser

If you want to change the session id on each log in, make sure to use session_regenerate_id(true) during the log in process.

<?php
session_start();
session_regenerate_id(true);
?> 

Iterate Multi-Dimensional Array with Nested Foreach Statement

I was looking for a solution to enumerate an array of an unknown at compile time rank with an access to every element indices set. I saw solutions with yield but here is another implementation with no yield. It is in old school minimalistic way. In this example AppendArrayDebug() just prints all the elements into StringBuilder buffer.

public static void AppendArrayDebug ( StringBuilder sb, Array array )
{
    if( array == null || array.Length == 0 )
    {
        sb.Append( "<nothing>" );
        return;
    }

    int i;

    var rank = array.Rank;
    var lastIndex = rank - 1;

    // Initialize indices and their boundaries
    var indices = new int[rank];
    var lower = new int[rank];
    var upper = new int[rank];
    for( i = 0; i < rank; ++i )
    {
        indices[i] = lower[i] = array.GetLowerBound( i );
        upper[i] = array.GetUpperBound( i );
    }

    while( true )
    {
        BeginMainLoop:

        // Begin work with an element

        var element = array.GetValue( indices );

        sb.AppendLine();
        sb.Append( '[' );
        for( i = 0; i < rank; ++i )
        {
            sb.Append( indices[i] );
            sb.Append( ' ' );
        }
        sb.Length -= 1;
        sb.Append( "] = " );
        sb.Append( element );

        // End work with the element

        // Increment index set

        // All indices except the first one are enumerated several times
        for( i = lastIndex; i > 0; )
        {
            if( ++indices[i] <= upper[i] )
                goto BeginMainLoop;
            indices[i] = lower[i];
            --i;
        }

        // Special case for the first index, it must be enumerated only once
        if( ++indices[0] > upper[0] )
            break;
    }
}

For example the following array will produce the following output:

var array = new [,,]
{
    { {  1,  2,  3 }, {  4,  5,  6 }, {  7,  8,  9 }, { 10, 11, 12 } },
    { { 13, 14, 15 }, { 16, 17, 18 }, { 19, 20, 21 }, { 22, 23, 24 } }
};

/*
Output:

[0 0 0] = 1
[0 0 1] = 2
[0 0 2] = 3
[0 1 0] = 4
[0 1 1] = 5
[0 1 2] = 6
[0 2 0] = 7
[0 2 1] = 8
[0 2 2] = 9
[0 3 0] = 10
[0 3 1] = 11
[0 3 2] = 12
[1 0 0] = 13
[1 0 1] = 14
[1 0 2] = 15
[1 1 0] = 16
[1 1 1] = 17
[1 1 2] = 18
[1 2 0] = 19
[1 2 1] = 20
[1 2 2] = 21
[1 3 0] = 22
[1 3 1] = 23
[1 3 2] = 24
*/

Asp.Net MVC with Drop Down List, and SelectListItem Assistance

Step-1: Your Model class

public class RechargeMobileViewModel
    {
        public string CustomerFullName { get; set; }
        public string TelecomSubscriber { get; set; }
        public int TotalAmount { get; set; }
        public string MobileNumber { get; set; }
        public int Month { get; set; }
        public List<SelectListItem> getAllDaysList { get; set; }

        // Define the list which you have to show in Drop down List
        public List<SelectListItem> getAllWeekDaysList()
        {
            List<SelectListItem> myList = new List<SelectListItem>();
            var data = new[]{
                 new SelectListItem{ Value="1",Text="Monday"},
                 new SelectListItem{ Value="2",Text="Tuesday"},
                 new SelectListItem{ Value="3",Text="Wednesday"},
                 new SelectListItem{ Value="4",Text="Thrusday"},
                 new SelectListItem{ Value="5",Text="Friday"},
                 new SelectListItem{ Value="6",Text="Saturday"},
                 new SelectListItem{ Value="7",Text="Sunday"},
             };
            myList = data.ToList();
            return myList;
        }
}

Step-2: Call this method to fill Drop down in your controller Action

namespace MvcVariousApplication.Controllers
    {
        public class HomeController : Controller
        {
            public ActionResult Index()
            {
              RechargeMobileViewModel objModel = new RechargeMobileViewModel();
                objModel.getAllDaysList = objModel.getAllWeekDaysList();  
                return View(objModel);
            }
    }
    }

Step-3: Fill your Drop-Down List of View as follows

 @model MvcVariousApplication.Models.RechargeMobileViewModel
    @{
        ViewBag.Title = "Contact";
    }
    @Html.LabelFor(model=> model.CustomerFullName)
    @Html.TextBoxFor(model => model.CustomerFullName)

    @Html.LabelFor(model => model.MobileNumber)
    @Html.TextBoxFor(model => model.MobileNumber)

    @Html.LabelFor(model => model.TelecomSubscriber)
    @Html.TextBoxFor(model => model.TelecomSubscriber)

    @Html.LabelFor(model => model.TotalAmount)
    @Html.TextBoxFor(model => model.TotalAmount)

    @Html.LabelFor(model => model.Month)
    @Html.DropDownListFor(model => model.Month, new SelectList(Model.getAllDaysList, "Value", "Text"), "-Select Day-")

Insert Data Into Tables Linked by Foreign Key

You can do it in one sql statement for existing customers, 3 statements for new ones. All you have to do is be an optimist and act as though the customer already exists:

insert into "order" (customer_id, price) values \
((select customer_id from customer where name = 'John'), 12.34);

If the customer does not exist, you'll get an sql exception which text will be something like:

null value in column "customer_id" violates not-null constraint

(providing you made customer_id non-nullable, which I'm sure you did). When that exception occurs, insert the customer into the customer table and redo the insert into the order table:

insert into customer(name) values ('John');
insert into "order" (customer_id, price) values \
((select customer_id from customer where name = 'John'), 12.34);

Unless your business is growing at a rate that will make "where to put all the money" your only real problem, most of your inserts will be for existing customers. So, most of the time, the exception won't occur and you'll be done in one statement.

Which is faster: multiple single INSERTs or one multiple-row INSERT?

Send as many inserts across the wire at one time as possible. The actual insert speed should be the same, but you will see performance gains from the reduction of network overhead.

Changing ImageView source

Or try this one. For me it's working fine:

imageView.setImageDrawable(ContextCompat.getDrawable(this, image));

Why is Visual Studio 2010 not able to find/open PDB files?

First change the following parameters:

Tools -> Options -> Debugging -> Symbols -> Server -> Yes

Then press Ctrl+F5 and you will see amazing things.

Can an Option in a Select tag carry multiple values?

Use a delimiter to separate the values.

<select name="myValues">
<option value="one|two">
</select>

<?php>
$value = filter_input(INPUT_POST, 'myValues');
$exploded_value = explode('|', $value);
$value_one = $exploded_value[0];
$value_two = $exploded_value[1];
?>

How do I write outputs to the Log in Android?

The Tag is just used to easily find your output, because the Output of LogCat can be sometimes very long. You can define somewhere in your class:

private static final String TAG = "myApp";

and use it when debugging

Log.v(TAG, "did something");

enter image description here

You can apply as well a Filter to only search for the tag.

Invisible characters - ASCII

There is actually a truly invisible character: U+FEFF. This character is called the Byte Order Mark and is related to the Unicode 8 system. It is a really confusing concept that can be explained HERE The Byte Order Mark or BOM for short is an invisible character that doesn't take up any space. You can copy the character bellow between the > and <.

Here is the character:

> <

How to catch this character in action:

  • Copy the character between the > and <,
  • Write a line of text, then randomly put your caret in the line of text
  • Paste the character in the line.
  • Go to the beginning of the line and press and hold the right arrow key.

You will notice that when your caret gets to the place you pasted the character, it will briefly stop for around half a second. This is becuase the caret is passing over the invisible character. Even though you can't see it doesn't mean it isn't there. The caret still sees that there is a character in that area that you pasted the BOM and will pass through it. Since the BOM is invisble, the caret will look like it has paused for a brief moment. You can past the BOM multiple times in an area and redo the steps above to really show the affect. Good luck!

EDIT: Sadly, Stackoverflow doesn't like the character. Here is an example from w3.org: https://www.w3.org/International/questions/examples/phpbomtest.php

JavaScript URL Decode function

If you are responsible for encoding the data in PHP using urlencode, PHP's rawurlencode works with JavaScript's decodeURIComponent without needing to replace the + character.

JavaScript: Create and save file

StreamSaver is an alternative to save very large files without having to keep all data in the memory.
In fact it emulates everything the server dose when saving a file but all client side with service worker.

You can either get the writer and manually write Uint8Array's to it or pipe a binary readableStream to the writable stream

There is a few example showcasing:

  • How to save multiple files as a zip
  • piping a readableStream from eg Response or blob.stream() to StreamSaver
  • manually writing to the writable stream as you type something
  • or recoding a video/audio

Here is an example in it's simplest form:

const fileStream = streamSaver.createWriteStream('filename.txt')

new Response('StreamSaver is awesome').body
  .pipeTo(fileStream)
  .then(success, error)

If you want to save a blob you would just convert that to a readableStream

new Response(blob).body.pipeTo(...) // response hack
blob.stream().pipeTo(...) // feature reference

How to start an Intent by passing some parameters to it?

I think you want something like this:

Intent foo = new Intent(this, viewContacts.class);
foo.putExtra("myFirstKey", "myFirstValue");
foo.putExtra("mySecondKey", "mySecondValue");
startActivity(foo);

or you can combine them into a bundle first. Corresponding getExtra() routines exist for the other side. See the intent topic in the dev guide for more information.

How do I prevent the error "Index signature of object type implicitly has an 'any' type" when compiling typescript with noImplicitAny flag enabled?

Declare the object like this.

export interface Thread {
    id:number;
    messageIds: number[];
    participants: {
        [key:number]: number
    };
}

How to install JQ on Mac by command-line?

The simplest way to install jq and test that it works is through brew and then using the simplest filter that merely formats the JSON

Install

brew is the easiest way to manage packages on a mac:

brew install jq

Need brew? Run the following command:

/usr/bin/ruby -e "$(curl -fsSL https://raw.githubusercontent.com/Homebrew/install/master/install)"

Failing that: instructions to install and use are on https://brew.sh/

Test

The . filter takes its input and produces it unchanged as output. This is the identity operator. (quote the docs)

echo '{ "name":"John", "age":31, "city":"New York" }' | jq .

The result should appear like so in your terminal:

{
  "name": "John",
  "age": 31,
  "city": "New York"
}

jQuery removing '-' character from string

If you want to remove all - you can use:

.replace(new RegExp('-', 'g'),"")

Looping over arrays, printing both index and value

In bash 4, you can use associative arrays:

declare -A foo
foo[0]="bar"
foo[35]="baz"
for key in "${!foo[@]}"
do
    echo "key: $key, value: ${foo[$key]}"
done

# output
# $ key: 0, value bar.
# $ key: 35, value baz.

In bash 3, this works (also works in zsh):

map=( )
map+=("0:bar")
map+=("35:baz")

for keyvalue in "${map[@]}" ; do
    key=${keyvalue%%:*}
    value=${keyvalue#*:}
    echo "key: $key, value $value."
done

How to open a second activity on click of button in android app

You can move to desired activity on button click. just add
android:onClick="timerApp"this line.

xml:

 <Button
        android:id="@+id/timer_app"
        android:layout_width="wrap_content"
        android:layout_height="wrap_content"
        android:onClick="timerApp"
        android:text="Click To run Timer Activity" />

In your main activity just add this method:

 public void timerApp(View view){
        Intent intent= new Intent(MainActivity.this,TimerActivity.class);
        startActivity(intent);
    }

OR in onCreate() method add below code

Button btn =findViewById(R.id.timer_app);//Don't need to type casting in android studio 3

btn.setOnClickListener(new View.OnClickListener() {
    public void onClick(View v) {
        Intent intent = new Intent(MainActivity.this, TimerActivity.class);
        startActivity(intent);       
    }  
});

jQuery: find element by text

Rocket's answer doesn't work.

<div>hhhhhh
<div>This is a test</div>
<div>Another Div</div>
</div>

I simply modified his DEMO here and you can see the root DOM is selected.

$('div:contains("test"):last').css('background-color', 'red');

add ":last" selector in the code to fix this.

Understanding dispatch_async

The main reason you use the default queue over the main queue is to run tasks in the background.

For instance, if I am downloading a file from the internet and I want to update the user on the progress of the download, I will run the download in the priority default queue and update the UI in the main queue asynchronously.

dispatch_async(dispatch_get_global_queue( DISPATCH_QUEUE_PRIORITY_DEFAULT, 0), ^(void){
    //Background Thread
    dispatch_async(dispatch_get_main_queue(), ^(void){
        //Run UI Updates
    });
});

How can you speed up Eclipse?

Close any open projects which are not in current use.

Try to switch off the auto publish mode during development.

Return Max Value of range that is determined by an Index & Match lookup

You can easily change the match-type to 1 when you are looking for the greatest value or to -1 when looking for the smallest value.

Unsetting array values in a foreach loop

$image is in your case the value of the item and not the key. Use the following syntax to get the key too:

foreach ($images as $key => $value) {
    /* … */
}

Now you can delete the item with unset($images[$key]).

Remote JMX connection

I have the same issue and I change any hostname that matches the local host name to 0.0.0.0, it seems to work after I do that.

SQL Server find and replace specific word in all rows of specific column

UPDATE tblKit
SET number = REPLACE(number, 'KIT', 'CH')
WHERE number like 'KIT%'

or simply this if you are sure that you have no values like this CKIT002

UPDATE tblKit
SET number = REPLACE(number, 'KIT', 'CH')

Drawing Circle with OpenGL

It looks like immediately after you draw the circle, you go into the main glut loop, where you've set the Draw() function to draw every time through the loop. So it's probably drawing the circle, then erasing it immediately and drawing the square. You should probably either make DrawCircle() your glutDisplayFunc(), or call DrawCircle() from Draw().

Match whitespace but not newlines

A variation on Greg’s answer that includes carriage returns too:

/[^\S\r\n]/

This regex is safer than /[^\S\n]/ with no \r. My reasoning is that Windows uses \r\n for newlines, and Mac OS 9 used \r. You’re unlikely to find \r without \n nowadays, but if you do find it, it couldn’t mean anything but a newline. Thus, since \r can mean a newline, we should exclude it too.

Stopping a windows service when the stop option is grayed out

sc queryex <service name>
taskkill /F /PID <Service PID>

eg

enter image description here

eg

Opening a remote machine's Windows C drive

If it's not the Home edition of XP, you can use \\servername\c$

Mark Brackett's comment:

Note that you need to be an Administrator on the local machine, as the share permissions are locked down

HTTP POST with URL query parameters -- good idea or not?

It would be fine to use query parameters on a POST endpoint, provided they refer to already existing resources.

For example:

POST /user_settings?user_id=4
{
  "use_safe_mode": 1
}

The POST above has a query parameter referring to an existing resource. The body parameter defines the new resource to be created.

(Granted, this may be more of a personal preference than a dogmatic principle.)

Find Item in ObservableCollection without using a loop

i had to use it for a condition add if you don't need the index

using System.Linq;

use

if(list.Any(x => x.Title == title){
// do something here
}

this will tell you if any variable satisfies your given condition.

ASP.net Getting the error "Access to the path is denied." while trying to upload files to my Windows Server 2008 R2 Web server

If anyone stumbles across this as it is the first result in google,

remember to specify the filename too in the SaveAs method.

Won't work

file_upload.PostedFile.SaveAs(Server.MapPath(SaveLocation));

You need this:

filename = Path.GetFileName(file_upload.PostedFile.FileName);
file_upload.PostedFile.SaveAs(Server.MapPath(SaveLocation + "\\" + filename));

I assumed the SaveAs method will automatically use the filename uploaded. Kept getting "Access denied" error. Not very descriptive of the actual problem

Char array to hex string C++

Code snippet above provides incorrect byte order in string, so I fixed it a bit.

char const hex[16] = { '0', '1', '2', '3', '4', '5', '6', '7', '8', '9', 'A',   'B','C','D','E','F'};

std::string byte_2_str(char* bytes, int size) {
  std::string str;
  for (int i = 0; i < size; ++i) {
    const char ch = bytes[i];
    str.append(&hex[(ch  & 0xF0) >> 4], 1);
    str.append(&hex[ch & 0xF], 1);
  }
  return str;
}

How to draw a path on a map using kml file?

Thank Mathias Lin, tested and it works!

In addition, sample implementation of Mathias's method in activity can be as follows.

public class DirectionMapActivity extends MapActivity {

    @Override
    public void onCreate(Bundle savedInstanceState) {
        super.onCreate(savedInstanceState);
        setContentView(R.layout.directionmap);

        MapView mapView = (MapView) findViewById(R.id.mapview);
        mapView.setBuiltInZoomControls(true);

        // Acquire a reference to the system Location Manager
        LocationManager locationManager = (LocationManager) this.getSystemService(Context.LOCATION_SERVICE);

        String locationProvider = LocationManager.NETWORK_PROVIDER;
        Location lastKnownLocation = locationManager.getLastKnownLocation(locationProvider);

        StringBuilder urlString = new StringBuilder();
        urlString.append("http://maps.google.com/maps?f=d&hl=en");
        urlString.append("&saddr=");//from
        urlString.append( Double.toString(lastKnownLocation.getLatitude() ));
        urlString.append(",");
        urlString.append( Double.toString(lastKnownLocation.getLongitude() ));
        urlString.append("&daddr=");//to
        urlString.append( Double.toString((double)dest[0]/1.0E6 ));
        urlString.append(",");
        urlString.append( Double.toString((double)dest[1]/1.0E6 ));
        urlString.append("&ie=UTF8&0&om=0&output=kml");

        try{
            // setup the url
            URL url = new URL(urlString.toString());
            // create the factory
            SAXParserFactory factory = SAXParserFactory.newInstance();
            // create a parser
            SAXParser parser = factory.newSAXParser();
            // create the reader (scanner)
            XMLReader xmlreader = parser.getXMLReader();
            // instantiate our handler
            NavigationSaxHandler navSaxHandler = new NavigationSaxHandler();
            // assign our handler
            xmlreader.setContentHandler(navSaxHandler);
            // get our data via the url class
            InputSource is = new InputSource(url.openStream());
            // perform the synchronous parse           
            xmlreader.parse(is);
            // get the results - should be a fully populated RSSFeed instance, or null on error
            NavigationDataSet ds = navSaxHandler.getParsedData();

            // draw path
            drawPath(ds, Color.parseColor("#add331"), mapView );

            // find boundary by using itemized overlay
            GeoPoint destPoint = new GeoPoint(dest[0],dest[1]);
            GeoPoint currentPoint = new GeoPoint( new Double(lastKnownLocation.getLatitude()*1E6).intValue()
                                                ,new Double(lastKnownLocation.getLongitude()*1E6).intValue() );

            Drawable dot = this.getResources().getDrawable(R.drawable.pixel);
            MapItemizedOverlay bgItemizedOverlay = new MapItemizedOverlay(dot,this);
            OverlayItem currentPixel = new OverlayItem(destPoint, null, null );
            OverlayItem destPixel = new OverlayItem(currentPoint, null, null );
            bgItemizedOverlay.addOverlay(currentPixel);
            bgItemizedOverlay.addOverlay(destPixel);

            // center and zoom in the map
            MapController mc = mapView.getController();
            mc.zoomToSpan(bgItemizedOverlay.getLatSpanE6()*2,bgItemizedOverlay.getLonSpanE6()*2);
            mc.animateTo(new GeoPoint(
                    (currentPoint.getLatitudeE6() + destPoint.getLatitudeE6()) / 2
                    , (currentPoint.getLongitudeE6() + destPoint.getLongitudeE6()) / 2));

        } catch(Exception e) {
            Log.d("DirectionMap","Exception parsing kml.");
        }

    }
    // and the rest of the methods in activity, e.g. drawPath() etc...

MapItemizedOverlay.java

public class MapItemizedOverlay extends ItemizedOverlay{
    private ArrayList<OverlayItem> mOverlays = new ArrayList<OverlayItem>();
    private Context mContext;

    public MapItemizedOverlay(Drawable defaultMarker, Context context) {
          super(boundCenterBottom(defaultMarker));
          mContext = context;
    }

    public void addOverlay(OverlayItem overlay) {
        mOverlays.add(overlay);
        populate();
    }

    @Override
    protected OverlayItem createItem(int i) {
      return mOverlays.get(i);
    }

    @Override
    public int size() {
      return mOverlays.size();
    }

}

How to sort pandas data frame using values from several columns?

DataFrame.sort is deprecated; use DataFrame.sort_values.

>>> df.sort_values(['c1','c2'], ascending=[False,True])
   c1   c2
0   3   10
3   2   15
1   2   30
4   2  100
2   1   20
>>> df.sort(['c1','c2'], ascending=[False,True])
Traceback (most recent call last):
  File "<stdin>", line 1, in <module>
  File "/Users/ampawake/anaconda/envs/pseudo/lib/python2.7/site-packages/pandas/core/generic.py", line 3614, in __getattr__
    return object.__getattribute__(self, name)
AttributeError: 'DataFrame' object has no attribute 'sort'

How to express a One-To-Many relationship in Django

To handle One-To-Many relationships in Django you need to use ForeignKey.

The documentation on ForeignKey is very comprehensive and should answer all the questions you have:

https://docs.djangoproject.com/en/dev/ref/models/fields/#foreignkey

The current structure in your example allows each Dude to have one number, and each number to belong to multiple Dudes (same with Business).

If you want the reverse relationship, you would need to add two ForeignKey fields to your PhoneNumber model, one to Dude and one to Business. This would allow each number to belong to either one Dude or one Business, and have Dudes and Businesses able to own multiple Numbers. I think this might be what you are after.

class Business(models.Model):
    ...
class Dude(models.Model):
    ...
class PhoneNumber(models.Model):
    dude = models.ForeignKey(Dude)
    business = models.ForeignKey(Business)

Passing arguments to JavaScript function from code-behind

Some other things I found out:

You can't directly pass in an array like:

this.Page.ClientScript.RegisterClientScriptBlock(this.GetType(), "xx",   
"<script>test("+x+","+y+");</script>");

because that calls the ToString() methods of x and y, which returns "System.Int32[]", and obviously Javascript can't use that. I had to pass in the arrays as strings, like "[1,2,3,4,5]", so I wrote a helper method to do the conversion.

Also, there is a difference between this.Page.ClientScript.RegisterStartupScript() and this.Page.ClientScript.RegisterClientScriptBlock() - the former places the script at the bottom of the page, which I need in order to be able to access the controls (like with document.getElementByID). RegisterClientScriptBlock() is executed before the tags are rendered, so I actually get a Javascript error if I use that method.

http://www.wrox.com/WileyCDA/Section/Manipulating-ASP-NET-Pages-and-Server-Controls-with-JavaScript.id-310803.html covers the difference between the two pretty well.

Here's the complete example I came up with:

// code behind
protected void Button1_Click(object sender, EventArgs e)
{
    int[] x = new int[] { 1, 2, 3, 4, 5 };
    int[] y = new int[] { 1, 2, 3, 4, 5 };

    string xStr = getArrayString(x); // converts {1,2,3,4,5} to [1,2,3,4,5]
    string yStr = getArrayString(y);

    string script = String.Format("test({0},{1})", xStr, yStr);
    this.Page.ClientScript.RegisterStartupScript(this.GetType(),
    "testFunction", script, true);
    //this.Page.ClientScript.RegisterClientScriptBlock(this.GetType(),
    //"testFunction", script, true); // different result
}
private string getArrayString(int[] array)
{
    StringBuilder sb = new StringBuilder();
    for (int i = 0; i < array.Length; i++)
    {
        sb.Append(array[i] + ",");
    }
    string arrayStr = string.Format("[{0}]", sb.ToString().TrimEnd(','));
    return arrayStr;
}

//aspx page
<html xmlns="http://www.w3.org/1999/xhtml">
<head runat="server">
    <title>Untitled Page</title>
    <script type="text/javascript">
    function test(x, y)
    {
        var text1 = document.getElementById("text1")
        for(var i = 0; i<x.length; i++)
        {
            text1.innerText += x[i]; // prints 12345
        }
        text1.innerText += "\ny: " + y; // prints y: 1,2,3,4,5

    }

    </script>
</head>
<body>
    <form id="form1" runat="server">
    <div>
        <asp:Button ID="Button1" runat="server" Text="Button"
         onclick="Button1_Click" />
    </div>
    <div id ="text1"> 
    </div>
    </form>
</body>
</html>

Any way to declare an array in-line?

Other option is to use ArrayUtils.toArray in org.apache.commons.lang3

ArrayUtils.toArray("elem1","elem2")

How to check if a column exists in Pandas

To check if one or more columns all exist, you can use set.issubset, as in:

if set(['A','C']).issubset(df.columns):
   df['sum'] = df['A'] + df['C']                

As @brianpck points out in a comment, set([]) can alternatively be constructed with curly braces,

if {'A', 'C'}.issubset(df.columns):

See this question for a discussion of the curly-braces syntax.

Or, you can use a list comprehension, as in:

if all([item in df.columns for item in ['A','C']]):

Add shadow to custom shape on Android

The following worked for me: Just save as custom_shape.xml.

<?xml version="1.0" encoding="utf-8"?>
<layer-list xmlns:android="http://schemas.android.com/apk/res/android" >

    <!-- "shadow" -->
    <item>
        <shape android:shape="rectangle" >
           <solid android:color="#000000"/>
           <corners android:radius="12dp" />
        </shape>
    </item>


    <item android:bottom="3px">
        <shape android:shape="rectangle"> 
            <solid android:color="#90ffffff"/>
            <corners android:radius="12dp" />
         </shape>
    </item>

</layer-list>

How to fetch Java version using single line command in Linux

This is a slight variation, but PJW's solution didn't quite work for me:

java -version 2>&1 | head -n 1 | cut -d'"' -f2

just cut the string on the delimiter " (double quotes) and get the second field.

Difference between "\n" and Environment.NewLine

Exact implementation of Environment.NewLine from the source code:

The implementation in .NET 4.6.1:

/*===================================NewLine====================================
**Action: A property which returns the appropriate newline string for the given
**        platform.
**Returns: \r\n on Win32.
**Arguments: None.
**Exceptions: None.
==============================================================================*/
public static String NewLine {
    get {
        Contract.Ensures(Contract.Result<String>() != null);
        return "\r\n";
    }
}

source


The implementation in .NET Core:

/*===================================NewLine====================================
**Action: A property which returns the appropriate newline string for the
**        given platform.
**Returns: \r\n on Win32.
**Arguments: None.
**Exceptions: None.
==============================================================================*/
public static String NewLine {
    get {
        Contract.Ensures(Contract.Result() != null);
#if !PLATFORM_UNIX
        return "\r\n";
#else
        return "\n";
#endif // !PLATFORM_UNIX
    }
}

source (in System.Private.CoreLib)

public static string NewLine => "\r\n";

source (in System.Runtime.Extensions)

Restful API service

Lets say I want to start the service on an event - onItemClicked() of a button. The Receiver mechanism would not work in that case because :-
a) I passed the Receiver to the service (as in Intent extra) from onItemClicked()
b) Activity moves to the background. In onPause() I set the receiver reference within the ResultReceiver to null to avoid leaking the Activity.
c) Activity gets destroyed.
d) Activity gets created again. However at this point the Service will not be able to make a callback to the Activity as that receiver reference is lost.
The mechanism of a limited broadcast or a PendingIntent seems to be more usefull in such scenarios- refer to Notify activity from service

MySQL CREATE FUNCTION Syntax

You have to override your ; delimiter with something like $$ to avoid this kind of error.

After your function definition, you can set the delimiter back to ;.

This should work:

DELIMITER $$
CREATE FUNCTION F_Dist3D (x1 decimal, y1 decimal) 
RETURNS decimal
DETERMINISTIC
BEGIN 
  DECLARE dist decimal;
  SET dist = SQRT(x1 - y1);
  RETURN dist;
END$$
DELIMITER ;

How can I change the text color with jQuery?

Place the following in your jQuery mouseover event handler:

$(this).css('color', 'red');

To set both color and size at the same time:

$(this).css({ 'color': 'red', 'font-size': '150%' });

You can set any CSS attribute using the .css() jQuery function.

SQL Server 2000: How to exit a stored procedure?

Its because you have no BEGIN and END statements. You shouldn't be seeing the prints, or errors running this statement, only Statement Completed (or something like that).

Could not find a declaration file for module 'module-name'. '/path/to/module-name.js' implicitly has an 'any' type

Could not find a declaration file for module 'busboy'. 'f:/firebase-cloud- 
functions/functions/node_modules/busboy/lib/main.js' implicitly has an ‘any’ 
type.

Try `npm install @types/busboy` if it exists or add a new declaration (.d.ts) 
the file containing `declare module 'busboy';`

In my case it's solved: All you have to do is edit your TypeScript Config file (tsconfig.json) and add a new key-value pair as:

"noImplicitAny": false

How do I get a reference to the app delegate in Swift?

In Swift 3.0 you can get the appdelegate reference by

let appDelegate = UIApplication.shared.delegate as! AppDelegate

Setting a global PowerShell variable from a function where the global variable name is a variable passed to the function

@zdan. Good answer. I'd improve it like this...

I think that the closest you can come to a true return value in PowerShell is to use a local variable to pass the value and never to use return as it may be 'corrupted' by any manner of output situations

function CheckRestart([REF]$retval)
{
    # Some logic
    $retval.Value = $true
}
[bool]$restart = $false
CheckRestart( [REF]$restart)
if ( $restart )
{
    Restart-Computer -Force
}

The $restart variable is used either side of the call to the function CheckRestart making clear the scope of the variable. The return value can by convention be either the first or last parameter declared. I prefer last.

setting textColor in TextView in layout/main.xml main layout file not referencing colors.xml file. (It wants a #RRGGBB instead of @color/text_color)

You have a typo in your xml; it should be:

android:textColor="@color/text_color"

that's "@color" without the 's'.

Disable all dialog boxes in Excel while running VB script?

Have you tried using the ConflictResolution:=xlLocalSessionChanges parameter in the SaveAs method?

As so:

Public Sub example()
Application.DisplayAlerts = False
Application.EnableEvents = False

For Each element In sArray
    XLSMToXLSX(element)
Next element

Application.DisplayAlerts = False
Application.EnableEvents = False
End Sub

Sub XLSMToXLSX(ByVal file As String)
    Do While WorkFile <> ""
        If Right(WorkFile, 4) <> "xlsx" Then
            Workbooks.Open Filename:=myPath & WorkFile

            Application.DisplayAlerts = False
            Application.EnableEvents = False

            ActiveWorkbook.SaveAs Filename:= _
            modifiedFileName, FileFormat:= _
            xlOpenXMLWorkbook, CreateBackup:=False, _
            ConflictResolution:=xlLocalSessionChanges

            Application.DisplayAlerts = True
            Application.EnableEvents = True

            ActiveWorkbook.Close
        End If
        WorkFile = Dir()
    Loop
End Sub

Multiple left-hand assignment with JavaScript

a = (b = 'string is truthy'); // b gets string; a gets b, which is a primitive (copy)
a = (b = { c: 'yes' }); // they point to the same object; a === b (not a copy)

(a && b) is logically (a ? b : a) and behaves like multiplication (eg. !!a * !!b)

(a || b) is logically (a ? a : b) and behaves like addition (eg. !!a + !!b)

(a = 0, b) is short for not caring if a is truthy, implicitly return b


a = (b = 0) && "nope, but a is 0 and b is 0"; // b is falsey + order of operations
a = (b = "b is this string") && "a gets this string"; // b is truthy + order of ops

JavaScript Operator Precedence (Order of Operations)

Note that the comma operator is actually the least privileged operator, but parenthesis are the most privileged, and they go hand-in-hand when constructing one-line expressions.


Eventually, you may need 'thunks' rather than hardcoded values, and to me, a thunk is both the function and the resultant value (the same 'thing').

const windowInnerHeight = () => 0.8 * window.innerHeight; // a thunk

windowInnerHeight(); // a thunk

Is putting a div inside an anchor ever correct?

The W3C doc doesn't use concepts like wrong and sin, but it does use those like provide the means, may be appropriate and discouraged.

Actually, in the second paragraph of section 4, the 4.01 spec itemizes its words as follows

The key words "MUST", "MUST NOT", "REQUIRED", "SHALL", "SHALL NOT", "SHOULD", "SHOULD NOT", "RECOMMENDED", "MAY", and "OPTIONAL" in this document are to be interpreted as described in [RFC2119]. However, for readability, these words do not appear in all uppercase letters in this specification.

With that in mind, I believe the definitive statement is in 7.5.3 Block-level and inline elements, where it says

Generally, inline elements may contain only data and other inline elements.

The condition "generally" appears to introduce enough ambiguity to say that HTML 4.01 does allow inline elements to contain block elements.

Certainly, CSS2 has a display property value, inline-block, that appears to be suited to the purpose you describe. I'm not sure if it was ever widely supported, but it seems that someone anticipated the need for that kind of behavior.

The DTD appear to be less forgiving here, but the text of the DTD defers to the spec:

The HTML 4.01 specification includes additional syntactic constraints that cannot be expressed within the DTDs.

In another comment, you suggest that you want to make a block active by wrapping it in an anchor. I don't believe HTML prohibits that, and CSS clearly allows it. So to answer the title question about whether it is ever correct, I say yes. By the standards, it is sometimes correct.

Making custom right-click context menus for my web-app

here is an example for right click context menu in javascript: Right Click Context Menu

Used raw javasScript Code for context menu functionality. Can you please check this, hope this will help you.

Live Code:

_x000D_
_x000D_
(function() {_x000D_
  _x000D_
  "use strict";_x000D_
_x000D_
_x000D_
  /*********************************************** Context Menu Function Only ********************************/_x000D_
  function clickInsideElement( e, className ) {_x000D_
    var el = e.srcElement || e.target;_x000D_
    if ( el.classList.contains(className) ) {_x000D_
      return el;_x000D_
    } else {_x000D_
      while ( el = el.parentNode ) {_x000D_
        if ( el.classList && el.classList.contains(className) ) {_x000D_
          return el;_x000D_
        }_x000D_
      }_x000D_
    }_x000D_
    return false;_x000D_
  }_x000D_
_x000D_
  function getPosition(e) {_x000D_
    var posx = 0, posy = 0;_x000D_
    if (!e) var e = window.event;_x000D_
    if (e.pageX || e.pageY) {_x000D_
      posx = e.pageX;_x000D_
      posy = e.pageY;_x000D_
    } else if (e.clientX || e.clientY) {_x000D_
      posx = e.clientX + document.body.scrollLeft + document.documentElement.scrollLeft;_x000D_
      posy = e.clientY + document.body.scrollTop + document.documentElement.scrollTop;_x000D_
    }_x000D_
    return {_x000D_
      x: posx,_x000D_
      y: posy_x000D_
    }_x000D_
  }_x000D_
_x000D_
  // Your Menu Class Name_x000D_
  var taskItemClassName = "thumb";_x000D_
  var contextMenuClassName = "context-menu",contextMenuItemClassName = "context-menu__item",contextMenuLinkClassName = "context-menu__link", contextMenuActive = "context-menu--active";_x000D_
  var taskItemInContext, clickCoords, clickCoordsX, clickCoordsY, menu = document.querySelector("#context-menu"), menuItems = menu.querySelectorAll(".context-menu__item");_x000D_
  var menuState = 0, menuWidth, menuHeight, menuPosition, menuPositionX, menuPositionY, windowWidth, windowHeight;_x000D_
_x000D_
  function initMenuFunction() {_x000D_
    contextListener();_x000D_
    clickListener();_x000D_
    keyupListener();_x000D_
    resizeListener();_x000D_
  }_x000D_
_x000D_
  /**_x000D_
   * Listens for contextmenu events._x000D_
   */_x000D_
  function contextListener() {_x000D_
    document.addEventListener( "contextmenu", function(e) {_x000D_
      taskItemInContext = clickInsideElement( e, taskItemClassName );_x000D_
_x000D_
      if ( taskItemInContext ) {_x000D_
        e.preventDefault();_x000D_
        toggleMenuOn();_x000D_
        positionMenu(e);_x000D_
      } else {_x000D_
        taskItemInContext = null;_x000D_
        toggleMenuOff();_x000D_
      }_x000D_
    });_x000D_
  }_x000D_
_x000D_
  /**_x000D_
   * Listens for click events._x000D_
   */_x000D_
  function clickListener() {_x000D_
    document.addEventListener( "click", function(e) {_x000D_
      var clickeElIsLink = clickInsideElement( e, contextMenuLinkClassName );_x000D_
_x000D_
      if ( clickeElIsLink ) {_x000D_
        e.preventDefault();_x000D_
        menuItemListener( clickeElIsLink );_x000D_
      } else {_x000D_
        var button = e.which || e.button;_x000D_
        if ( button === 1 ) {_x000D_
          toggleMenuOff();_x000D_
        }_x000D_
      }_x000D_
    });_x000D_
  }_x000D_
_x000D_
  /**_x000D_
   * Listens for keyup events._x000D_
   */_x000D_
  function keyupListener() {_x000D_
    window.onkeyup = function(e) {_x000D_
      if ( e.keyCode === 27 ) {_x000D_
        toggleMenuOff();_x000D_
      }_x000D_
    }_x000D_
  }_x000D_
_x000D_
  /**_x000D_
   * Window resize event listener_x000D_
   */_x000D_
  function resizeListener() {_x000D_
    window.onresize = function(e) {_x000D_
      toggleMenuOff();_x000D_
    };_x000D_
  }_x000D_
_x000D_
  /**_x000D_
   * Turns the custom context menu on._x000D_
   */_x000D_
  function toggleMenuOn() {_x000D_
    if ( menuState !== 1 ) {_x000D_
      menuState = 1;_x000D_
      menu.classList.add( contextMenuActive );_x000D_
    }_x000D_
  }_x000D_
_x000D_
  /**_x000D_
   * Turns the custom context menu off._x000D_
   */_x000D_
  function toggleMenuOff() {_x000D_
    if ( menuState !== 0 ) {_x000D_
      menuState = 0;_x000D_
      menu.classList.remove( contextMenuActive );_x000D_
    }_x000D_
  }_x000D_
_x000D_
  function positionMenu(e) {_x000D_
    clickCoords = getPosition(e);_x000D_
    clickCoordsX = clickCoords.x;_x000D_
    clickCoordsY = clickCoords.y;_x000D_
    menuWidth = menu.offsetWidth + 4;_x000D_
    menuHeight = menu.offsetHeight + 4;_x000D_
_x000D_
    windowWidth = window.innerWidth;_x000D_
    windowHeight = window.innerHeight;_x000D_
_x000D_
    if ( (windowWidth - clickCoordsX) < menuWidth ) {_x000D_
      menu.style.left = (windowWidth - menuWidth)-0 + "px";_x000D_
    } else {_x000D_
      menu.style.left = clickCoordsX-0 + "px";_x000D_
    }_x000D_
_x000D_
    // menu.style.top = clickCoordsY + "px";_x000D_
_x000D_
    if ( Math.abs(windowHeight - clickCoordsY) < menuHeight ) {_x000D_
      menu.style.top = (windowHeight - menuHeight)-0 + "px";_x000D_
    } else {_x000D_
      menu.style.top = clickCoordsY-0 + "px";_x000D_
    }_x000D_
  }_x000D_
_x000D_
_x000D_
  function menuItemListener( link ) {_x000D_
    var menuSelectedPhotoId = taskItemInContext.getAttribute("data-id");_x000D_
    console.log('Your Selected Photo: '+menuSelectedPhotoId)_x000D_
    var moveToAlbumSelectedId = link.getAttribute("data-action");_x000D_
    if(moveToAlbumSelectedId == 'remove'){_x000D_
      console.log('You Clicked the remove button')_x000D_
    }else if(moveToAlbumSelectedId && moveToAlbumSelectedId.length > 7){_x000D_
      console.log('Clicked Album Name: '+moveToAlbumSelectedId);_x000D_
    }_x000D_
    toggleMenuOff();_x000D_
  }_x000D_
  initMenuFunction();_x000D_
_x000D_
})();
_x000D_
/* For Body Padding and content */_x000D_
body { padding-top: 70px; }_x000D_
li a { text-decoration: none !important; }_x000D_
_x000D_
/* Thumbnail only */_x000D_
.thumb {_x000D_
  margin-bottom: 30px;_x000D_
}_x000D_
.thumb:hover a, .thumb:active a, .thumb:focus a {_x000D_
  border: 1px solid purple;_x000D_
}_x000D_
_x000D_
/************** For Context menu ***********/_x000D_
/* context menu */_x000D_
.context-menu {  display: none;  position: absolute;  z-index: 9999;  padding: 12px 0;  width: 200px;  background-color: #fff;  border: solid 1px #dfdfdf;  box-shadow: 1px 1px 2px #cfcfcf;  }_x000D_
.context-menu--active {  display: block;  }_x000D_
_x000D_
.context-menu__items { list-style: none;  margin: 0;  padding: 0;  }_x000D_
.context-menu__item { display: block;  margin-bottom: 4px;  }_x000D_
.context-menu__item:last-child {  margin-bottom: 0;  }_x000D_
.context-menu__link {  display: block;  padding: 4px 12px;  color: #0066aa;  text-decoration: none;  }_x000D_
.context-menu__link:hover {  color: #fff;  background-color: #0066aa;  }_x000D_
.context-menu__items ul {  position: absolute;  white-space: nowrap;  z-index: 1;  left: -99999em;}_x000D_
.context-menu__items > li:hover > ul {  left: auto;  padding-top: 5px  ;  min-width: 100%;  }_x000D_
.context-menu__items > li li ul {  border-left:1px solid #fff;}_x000D_
.context-menu__items > li li:hover > ul {  left: 100%;  top: -1px;  }_x000D_
.context-menu__item ul { background-color: #ffffff; padding: 7px 11px;  list-style-type: none;  text-decoration: none; margin-left: 40px; }_x000D_
.page-media .context-menu__items ul li { display: block; }_x000D_
/************** For Context menu ***********/
_x000D_
<link href="https://maxcdn.bootstrapcdn.com/font-awesome/4.7.0/css/font-awesome.min.css" rel="stylesheet"/>_x000D_
<link href="https://cdnjs.cloudflare.com/ajax/libs/twitter-bootstrap/3.3.7/css/bootstrap.min.css" rel="stylesheet"/>_x000D_
<body>_x000D_
_x000D_
_x000D_
_x000D_
    <!-- Page Content -->_x000D_
    <div class="container">_x000D_
_x000D_
        <div class="row">_x000D_
_x000D_
            <div class="col-lg-12">_x000D_
                <h1 class="page-header">Thumbnail Gallery <small>(Right click to see the context menu)</small></h1>_x000D_
            </div>_x000D_
_x000D_
            <div class="col-lg-3 col-md-4 col-xs-6 thumb">_x000D_
                <a class="thumbnail" href="#">_x000D_
                    <img class="img-responsive" src="http://placehold.it/400x300" alt="">_x000D_
                </a>_x000D_
            </div>_x000D_
            <div class="col-lg-3 col-md-4 col-xs-6 thumb">_x000D_
                <a class="thumbnail" href="#">_x000D_
                    <img class="img-responsive" src="http://placehold.it/400x300" alt="">_x000D_
                </a>_x000D_
            </div>_x000D_
            <div class="col-lg-3 col-md-4 col-xs-6 thumb">_x000D_
                <a class="thumbnail" href="#">_x000D_
                    <img class="img-responsive" src="http://placehold.it/400x300" alt="">_x000D_
                </a>_x000D_
            </div>_x000D_
            <div class="col-lg-3 col-md-4 col-xs-6 thumb">_x000D_
                <a class="thumbnail" href="#">_x000D_
                    <img class="img-responsive" src="http://placehold.it/400x300" alt="">_x000D_
                </a>_x000D_
            </div>_x000D_
            <div class="col-lg-3 col-md-4 col-xs-6 thumb">_x000D_
                <a class="thumbnail" href="#">_x000D_
                    <img class="img-responsive" src="http://placehold.it/400x300" alt="">_x000D_
                </a>_x000D_
            </div>_x000D_
            <div class="col-lg-3 col-md-4 col-xs-6 thumb">_x000D_
                <a class="thumbnail" href="#">_x000D_
                    <img class="img-responsive" src="http://placehold.it/400x300" alt="">_x000D_
                </a>_x000D_
            </div>_x000D_
            <div class="col-lg-3 col-md-4 col-xs-6 thumb">_x000D_
                <a class="thumbnail" href="#">_x000D_
                    <img class="img-responsive" src="http://placehold.it/400x300" alt="">_x000D_
                </a>_x000D_
            </div>_x000D_
            <div class="col-lg-3 col-md-4 col-xs-6 thumb">_x000D_
                <a class="thumbnail" href="#">_x000D_
                    <img class="img-responsive" src="http://placehold.it/400x300" alt="">_x000D_
                </a>_x000D_
            </div>_x000D_
_x000D_
        </div>_x000D_
_x000D_
        <hr>_x000D_
_x000D_
_x000D_
    </div>_x000D_
    <!-- /.container -->_x000D_
_x000D_
_x000D_
    <!-- / The Context Menu -->_x000D_
    <nav id="context-menu" class="context-menu">_x000D_
        <ul class="context-menu__items">_x000D_
            <li class="context-menu__item">_x000D_
                <a href="#" class="context-menu__link" data-action="Delete This Photo"><i class="fa fa-empire"></i> Delete This Photo</a>_x000D_
            </li>_x000D_
            <li class="context-menu__item">_x000D_
                <a href="#" class="context-menu__link" data-action="Photo Option 2"><i class="fa fa-envira"></i> Photo Option 2</a>_x000D_
            </li>_x000D_
            <li class="context-menu__item">_x000D_
                <a href="#" class="context-menu__link" data-action="Photo Option 3"><i class="fa fa-first-order"></i> Photo Option 3</a>_x000D_
            </li>_x000D_
            <li class="context-menu__item">_x000D_
                <a href="#" class="context-menu__link" data-action="Photo Option 4"><i class="fa fa-gitlab"></i> Photo Option 4</a>_x000D_
            </li>_x000D_
            <li class="context-menu__item">_x000D_
                <a href="#" class="context-menu__link" data-action="Photo Option 5"><i class="fa fa-ioxhost"></i> Photo Option 5</a>_x000D_
            </li>_x000D_
            <li class="context-menu__item">_x000D_
                <a href="#" class="context-menu__link"><i class="fa fa-arrow-right"></i> Add Photo to</a>_x000D_
                <ul>_x000D_
                    <li><a href="#!" class="context-menu__link" data-action="album-one"><i class="fa fa-camera-retro"></i> Album One</a></li>_x000D_
                    <li><a href="#!" class="context-menu__link" data-action="album-two"><i class="fa fa-camera-retro"></i> Album Two</a></li>_x000D_
                    <li><a href="#!" class="context-menu__link" data-action="album-three"><i class="fa fa-camera-retro"></i> Album Three</a></li>_x000D_
                    <li><a href="#!" class="context-menu__link" data-action="album-four"><i class="fa fa-camera-retro"></i> Album Four</a></li>_x000D_
                </ul>_x000D_
            </li>_x000D_
        </ul>_x000D_
    </nav>_x000D_
_x000D_
    <!-- End # Context Menu -->_x000D_
_x000D_
_x000D_
</body>
_x000D_
_x000D_
_x000D_

JavaScript/jQuery: replace part of string?

You need to set the text after the replace call:

_x000D_
_x000D_
$('.element span').each(function() {_x000D_
  console.log($(this).text());_x000D_
  var text = $(this).text().replace('N/A, ', '');_x000D_
  $(this).text(text);_x000D_
});
_x000D_
<script src="https://cdnjs.cloudflare.com/ajax/libs/jquery/3.3.1/jquery.min.js"></script>_x000D_
<div class="element">_x000D_
  <span>N/A, Category</span>_x000D_
</div>
_x000D_
_x000D_
_x000D_


Here's another cool way you can do it (hat tip @Felix King):

$(".element span").text(function(index, text) {
    return text.replace("N/A, ", "");
});

Transparent image - background color

If I understand you right, you can do this:

<img src="image.png" style="background-color:red;" />

In fact, you can even apply a whole background-image to the image, resulting in two "layers" without the need for multi-background support in the browser ;)

LDAP Authentication using Java

This is my LDAP Java login test application supporting LDAP:// and LDAPS:// self-signed test certificate. Code is taken from few SO posts, simplified implementation and removed legacy sun.java.* imports.

Usage
I have run this in Windows7 and Linux machines against WinAD directory service. Application prints username and member groups.

$ java -cp classes test.LoginLDAP url=ldap://1.2.3.4:389 [email protected] password=mypwd

$ java -cp classes test.LoginLDAP url=ldaps://1.2.3.4:636 [email protected] password=mypwd

Test application supports temporary self-signed test certificates for ldaps:// protocol, this DummySSLFactory accepts any server cert so man-in-the-middle is possible. Real life installation should import server certificate to a local JKS keystore file and not using dummy factory.

Application uses enduser's username+password for initial context and ldap queries, it works for WinAD but don't know if can be used for all ldap server implementations. You could create context with internal username+pwd then run queries to see if given enduser is found.

LoginLDAP.java

package test;

import java.util.*;
import javax.naming.*;
import javax.naming.directory.*;

public class LoginLDAP {

    public static void main(String[] args) throws Exception {
        Map<String,String> params = createParams(args);

        String url = params.get("url"); // ldap://1.2.3.4:389 or ldaps://1.2.3.4:636
        String principalName = params.get("username"); // [email protected]
        String domainName = params.get("domain"); // mydomain.com or empty

        if (domainName==null || "".equals(domainName)) {
            int delim = principalName.indexOf('@');
            domainName = principalName.substring(delim+1);
        }

        Properties props = new Properties();
        props.put(Context.INITIAL_CONTEXT_FACTORY, "com.sun.jndi.ldap.LdapCtxFactory");
        props.put(Context.PROVIDER_URL, url); 
        props.put(Context.SECURITY_PRINCIPAL, principalName); 
        props.put(Context.SECURITY_CREDENTIALS, params.get("password")); // secretpwd
        if (url.toUpperCase().startsWith("LDAPS://")) {
            props.put(Context.SECURITY_PROTOCOL, "ssl");
            props.put(Context.SECURITY_AUTHENTICATION, "simple");
            props.put("java.naming.ldap.factory.socket", "test.DummySSLSocketFactory");         
        }

        InitialDirContext context = new InitialDirContext(props);
        try {
            SearchControls ctrls = new SearchControls();
            ctrls.setSearchScope(SearchControls.SUBTREE_SCOPE);
            NamingEnumeration<SearchResult> results = context.search(toDC(domainName),"(& (userPrincipalName="+principalName+")(objectClass=user))", ctrls);
            if(!results.hasMore())
                throw new AuthenticationException("Principal name not found");

            SearchResult result = results.next();
            System.out.println("distinguisedName: " + result.getNameInNamespace() ); // CN=Firstname Lastname,OU=Mycity,DC=mydomain,DC=com

            Attribute memberOf = result.getAttributes().get("memberOf");
            if(memberOf!=null) {
                for(int idx=0; idx<memberOf.size(); idx++) {
                    System.out.println("memberOf: " + memberOf.get(idx).toString() ); // CN=Mygroup,CN=Users,DC=mydomain,DC=com
                    //Attribute att = context.getAttributes(memberOf.get(idx).toString(), new String[]{"CN"}).get("CN");
                    //System.out.println( att.get().toString() ); //  CN part of groupname
                }
            }
        } finally {
            try { context.close(); } catch(Exception ex) { }
        }       
    }

    /**
     * Create "DC=sub,DC=mydomain,DC=com" string
     * @param domainName    sub.mydomain.com
     * @return
     */
    private static String toDC(String domainName) {
        StringBuilder buf = new StringBuilder();
        for (String token : domainName.split("\\.")) {
            if(token.length()==0) continue;
            if(buf.length()>0)  buf.append(",");
            buf.append("DC=").append(token);
        }
        return buf.toString();
    }

    private static Map<String,String> createParams(String[] args) {
        Map<String,String> params = new HashMap<String,String>();  
        for(String str : args) {
            int delim = str.indexOf('=');
            if (delim>0) params.put(str.substring(0, delim).trim(), str.substring(delim+1).trim());
            else if (delim==0) params.put("", str.substring(1).trim());
            else params.put(str, null);
        }
        return params;
    }

}

And SSL helper class.

package test;

import java.io.*;
import java.net.*;
import java.security.SecureRandom;
import java.security.cert.X509Certificate;    
import javax.net.*;
import javax.net.ssl.*;

public class DummySSLSocketFactory extends SSLSocketFactory {
    private SSLSocketFactory socketFactory;
    public DummySSLSocketFactory() {
        try {
          SSLContext ctx = SSLContext.getInstance("TLS");
          ctx.init(null, new TrustManager[]{ new DummyTrustManager()}, new SecureRandom());
          socketFactory = ctx.getSocketFactory();
        } catch ( Exception ex ){ throw new IllegalArgumentException(ex); }
    }

      public static SocketFactory getDefault() { return new DummySSLSocketFactory(); }

      @Override public String[] getDefaultCipherSuites() { return socketFactory.getDefaultCipherSuites(); }
      @Override public String[] getSupportedCipherSuites() { return socketFactory.getSupportedCipherSuites(); }

      @Override public Socket createSocket(Socket socket, String string, int i, boolean bln) throws IOException {
        return socketFactory.createSocket(socket, string, i, bln);
      }
      @Override public Socket createSocket(String string, int i) throws IOException, UnknownHostException {
        return socketFactory.createSocket(string, i);
      }
      @Override public Socket createSocket(String string, int i, InetAddress ia, int i1) throws IOException, UnknownHostException {
        return socketFactory.createSocket(string, i, ia, i1);
      }
      @Override public Socket createSocket(InetAddress ia, int i) throws IOException {
        return socketFactory.createSocket(ia, i);
      }
      @Override public Socket createSocket(InetAddress ia, int i, InetAddress ia1, int i1) throws IOException {
        return socketFactory.createSocket(ia, i, ia1, i1);
      }
}

class DummyTrustManager implements X509TrustManager {
    @Override public void checkClientTrusted(X509Certificate[] xcs, String str) {
        // do nothing
    }
    @Override public void checkServerTrusted(X509Certificate[] xcs, String str) {
        /*System.out.println("checkServerTrusted for authType: " + str); // RSA
        for(int idx=0; idx<xcs.length; idx++) {
            X509Certificate cert = xcs[idx];
            System.out.println("X500Principal: " + cert.getSubjectX500Principal().getName());
        }*/
    }
    @Override public X509Certificate[] getAcceptedIssuers() {
        return new java.security.cert.X509Certificate[0];
    }
}

Adding Table rows Dynamically in Android

You can use an inflater with TableRow:

for (int i = 0; i < months; i++) {
    
    View view = getLayoutInflater ().inflate (R.layout.list_month_data, null, false);
        
    TextView textView = view.findViewById (R.id.title);
    
    textView.setText ("Text");
        
    tableLayout.addView (view);
    
}

Layout:

<TableRow
    xmlns:android="http://schemas.android.com/apk/res/android"
    android:layout_width="match_parent"
    android:layout_height="wrap_content"
    android:orientation="vertical"
    android:layout_centerInParent="true"
    android:gravity="center_horizontal"
    android:paddingTop="15dp"
    android:paddingRight="15dp"
    android:paddingLeft="15dp"
    android:paddingBottom="10dp"
    >
    
    <TextView
        android:id="@+id/title"
        android:layout_width="wrap_content"
        android:layout_height="wrap_content"
        android:textSize="18sp"
        android:gravity="center"
        />
    
</TableRow>

Embedding DLLs in a compiled executable

If they're actually managed assemblies, you can use ILMerge. For native DLLs, you'll have a bit more work to do.

See also: How can a C++ windows dll be merged into a C# application exe?

How can I position my div at the bottom of its container?

There is nothing called float:bottom in CSS. The best way is using positioning in such cases:

position:absolute;
bottom:0;

Resolving IP Address from hostname with PowerShell

The Test-Connection command seems to be a useful alternative, and it can either provide either a Win32_PingStatus object, or a boolean value.

Documentation: https://msdn.microsoft.com/en-us/powershell/reference/5.1/microsoft.powershell.management/test-connection

SQL state [99999]; error code [17004]; Invalid column type: 1111 With Spring SimpleJdbcCall

I have a function which returns a CLOB and I was seeing the above error when I'd forgotten to declare the return value as an output parameter. Initially I had:

protected SimpleJdbcCall buildJdbcCall(JdbcTemplate jdbcTemplate)
{
    SimpleJdbcCall call = new SimpleJdbcCall(jdbcTemplate)
        .withSchemaName(schema)
        .withCatalogName(catalog)
        .withFunctionName(functionName)
        .withReturnValue()          
        .declareParameters(buildSqlParameters());

    return call;
}

public SqlParameter[] buildSqlParameters() {
    return new SqlParameter[]{
        new SqlParameter("p_names", Types.VARCHAR),
        new SqlParameter("p_format", Types.VARCHAR),
        new SqlParameter("p_units", Types.VARCHAR),
        new SqlParameter("p_datums", Types.VARCHAR),
        new SqlParameter("p_start", Types.VARCHAR),
        new SqlParameter("p_end", Types.VARCHAR),
        new SqlParameter("p_timezone", Types.VARCHAR),
        new SqlParameter("p_office_id", Types.VARCHAR),
        };
}

The buildSqlParameters method should have included the SqlOutParameter:

public SqlParameter[] buildSqlParameters() {
    return new SqlParameter[]{
        new SqlParameter("p_names", Types.VARCHAR),
        new SqlParameter("p_format", Types.VARCHAR),
        new SqlParameter("p_units", Types.VARCHAR),
        new SqlParameter("p_datums", Types.VARCHAR),
        new SqlParameter("p_start", Types.VARCHAR),
        new SqlParameter("p_end", Types.VARCHAR),
        new SqlParameter("p_timezone", Types.VARCHAR),
        new SqlParameter("p_office_id", Types.VARCHAR),
        new SqlOutParameter("l_clob", Types.CLOB)  // <-- This was missing!
    }; 
}

Angular bootstrap datepicker date format does not format ng-model value

I can fix this by adding below code in my JSP file. Now both model and UI values are same.

<div ng-show="false">
    {{dt = (dt | date:'dd-MMMM-yyyy') }}
</div>  

Using GitLab token to clone without authentication

One possible way is using a deploy token (https://docs.gitlab.com/ee/user/project/deploy_tokens). After creating the token, use:

git clone https://<username>:<deploy_token>@gitlab.example.com/tanuki/awesome_project.git 

as mentioned in the link above.

Where/How to getIntent().getExtras() in an Android Fragment?

you can still use

String Item = getIntent().getExtras().getString("name");

in the fragment, you just need call getActivity() first:

String Item = getActivity().getIntent().getExtras().getString("name");

This saves you having to write some code.

How do I base64 encode (decode) in C?

Small improvement to the code from ryyst (who got the most votes) is to not use dynamically allocated decoding table but rather static const precomputed table. This eliminates the use of pointer and initialization of the table, and also avoids memory leakage if one forgets to clean up the decoding table with base64_cleanup() (by the way, in base64_cleanup(), after calling free(decoding_table), one should have decoding_table=NULL, otherwise accidentally calling base64_decode after base64_cleanup() will crash or cause undetermined behavior). Another solution could be to use std::unique_ptr...but I'm satisfied with just having const char[256] on the stack and avoid using pointers alltogether - the code looks cleaner and shorter this way.

The decoding table is computed as follows:

const char encoding_table[] = { 
    'A', 'B', 'C', 'D', 'E', 'F', 'G', 'H',
    'I', 'J', 'K', 'L', 'M', 'N', 'O', 'P',
    'Q', 'R', 'S', 'T', 'U', 'V', 'W', 'X',
    'Y', 'Z', 'a', 'b', 'c', 'd', 'e', 'f',
    'g', 'h', 'i', 'j', 'k', 'l', 'm', 'n',
    'o', 'p', 'q', 'r', 's', 't', 'u', 'v',
    'w', 'x', 'y', 'z', '0', '1', '2', '3',
    '4', '5', '6', '7', '8', '9', '+', '/' };

unsigned char decoding_table[256];

for (int i = 0; i < 256; i++)
    decoding_table[i] = '\0';

for (int i = 0; i < 64; i++)
    decoding_table[(unsigned char)encoding_table[i]] = i;

for (int i = 0; i < 256; i++)
    cout << "0x" << (int(decoding_table[i]) < 16 ? "0" : "") << hex << int(decoding_table[i]) << (i != 255 ? "," : "") << ((i+1) % 16 == 0 ? '\n' : '\0');

cin.ignore();

and the modified code I am using is:

        static const char encoding_table[] = { 
            'A', 'B', 'C', 'D', 'E', 'F', 'G', 'H',
            'I', 'J', 'K', 'L', 'M', 'N', 'O', 'P',
            'Q', 'R', 'S', 'T', 'U', 'V', 'W', 'X',
            'Y', 'Z', 'a', 'b', 'c', 'd', 'e', 'f',
            'g', 'h', 'i', 'j', 'k', 'l', 'm', 'n',
            'o', 'p', 'q', 'r', 's', 't', 'u', 'v',
            'w', 'x', 'y', 'z', '0', '1', '2', '3',
            '4', '5', '6', '7', '8', '9', '+', '/' };

        static const unsigned char decoding_table[256] = {
            0x00, 0x00, 0x00, 0x00, 0x00, 0x00, 0x00, 0x00, 0x00, 0x00, 0x00, 0x00, 0x00, 0x00, 0x00, 0x00,
            0x00, 0x00, 0x00, 0x00, 0x00, 0x00, 0x00, 0x00, 0x00, 0x00, 0x00, 0x00, 0x00, 0x00, 0x00, 0x00,
            0x00, 0x00, 0x00, 0x00, 0x00, 0x00, 0x00, 0x00, 0x00, 0x00, 0x00, 0x3e, 0x00, 0x00, 0x00, 0x3f,
            0x34, 0x35, 0x36, 0x37, 0x38, 0x39, 0x3a, 0x3b, 0x3c, 0x3d, 0x00, 0x00, 0x00, 0x00, 0x00, 0x00,
            0x00, 0x00, 0x01, 0x02, 0x03, 0x04, 0x05, 0x06, 0x07, 0x08, 0x09, 0x0a, 0x0b, 0x0c, 0x0d, 0x0e,
            0x0f, 0x10, 0x11, 0x12, 0x13, 0x14, 0x15, 0x16, 0x17, 0x18, 0x19, 0x00, 0x00, 0x00, 0x00, 0x00,
            0x00, 0x1a, 0x1b, 0x1c, 0x1d, 0x1e, 0x1f, 0x20, 0x21, 0x22, 0x23, 0x24, 0x25, 0x26, 0x27, 0x28,
            0x29, 0x2a, 0x2b, 0x2c, 0x2d, 0x2e, 0x2f, 0x30, 0x31, 0x32, 0x33, 0x00, 0x00, 0x00, 0x00, 0x00,
            0x00, 0x00, 0x00, 0x00, 0x00, 0x00, 0x00, 0x00, 0x00, 0x00, 0x00, 0x00, 0x00, 0x00, 0x00, 0x00,
            0x00, 0x00, 0x00, 0x00, 0x00, 0x00, 0x00, 0x00, 0x00, 0x00, 0x00, 0x00, 0x00, 0x00, 0x00, 0x00,
            0x00, 0x00, 0x00, 0x00, 0x00, 0x00, 0x00, 0x00, 0x00, 0x00, 0x00, 0x00, 0x00, 0x00, 0x00, 0x00,
            0x00, 0x00, 0x00, 0x00, 0x00, 0x00, 0x00, 0x00, 0x00, 0x00, 0x00, 0x00, 0x00, 0x00, 0x00, 0x00,
            0x00, 0x00, 0x00, 0x00, 0x00, 0x00, 0x00, 0x00, 0x00, 0x00, 0x00, 0x00, 0x00, 0x00, 0x00, 0x00,
            0x00, 0x00, 0x00, 0x00, 0x00, 0x00, 0x00, 0x00, 0x00, 0x00, 0x00, 0x00, 0x00, 0x00, 0x00, 0x00,
            0x00, 0x00, 0x00, 0x00, 0x00, 0x00, 0x00, 0x00, 0x00, 0x00, 0x00, 0x00, 0x00, 0x00, 0x00, 0x00,
            0x00, 0x00, 0x00, 0x00, 0x00, 0x00, 0x00, 0x00, 0x00, 0x00, 0x00, 0x00, 0x00, 0x00, 0x00, 0x00 };

        char* base64_encode(const unsigned char *data, size_t input_length, size_t &output_length) {

            const int mod_table[] = { 0, 2, 1 };

            output_length = 4 * ((input_length + 2) / 3);

            char *encoded_data = (char*)malloc(output_length);

            if (encoded_data == nullptr)
                return nullptr;

            for (int i = 0, j = 0; i < input_length;) {

                uint32_t octet_a = i < input_length ? (unsigned char)data[i++] : 0;
                uint32_t octet_b = i < input_length ? (unsigned char)data[i++] : 0;
                uint32_t octet_c = i < input_length ? (unsigned char)data[i++] : 0;

                uint32_t triple = (octet_a << 0x10) + (octet_b << 0x08) + octet_c;

                encoded_data[j++] = encoding_table[(triple >> 3 * 6) & 0x3F];
                encoded_data[j++] = encoding_table[(triple >> 2 * 6) & 0x3F];
                encoded_data[j++] = encoding_table[(triple >> 1 * 6) & 0x3F];
                encoded_data[j++] = encoding_table[(triple >> 0 * 6) & 0x3F];

            }

            for (int i = 0; i < mod_table[input_length % 3]; i++)
                encoded_data[output_length - 1 - i] = '=';

            return encoded_data;

        };

        unsigned char* base64_decode(const char *data, size_t input_length, size_t &output_length) {        

            if (input_length % 4 != 0)
                return nullptr;

            output_length = input_length / 4 * 3;

            if (data[input_length - 1] == '=') (output_length)--;
            if (data[input_length - 2] == '=') (output_length)--;

            unsigned char* decoded_data = (unsigned char*)malloc(output_length);

            if (decoded_data == nullptr)
                return nullptr;

            for (int i = 0, j = 0; i < input_length;) {

                uint32_t sextet_a = data[i] == '=' ? 0 & i++ : decoding_table[data[i++]];
                uint32_t sextet_b = data[i] == '=' ? 0 & i++ : decoding_table[data[i++]];
                uint32_t sextet_c = data[i] == '=' ? 0 & i++ : decoding_table[data[i++]];
                uint32_t sextet_d = data[i] == '=' ? 0 & i++ : decoding_table[data[i++]];

                uint32_t triple = (sextet_a << 3 * 6)
                    + (sextet_b << 2 * 6)
                    + (sextet_c << 1 * 6)
                    + (sextet_d << 0 * 6);

                if (j < output_length) decoded_data[j++] = (triple >> 2 * 8) & 0xFF;
                if (j < output_length) decoded_data[j++] = (triple >> 1 * 8) & 0xFF;
                if (j < output_length) decoded_data[j++] = (triple >> 0 * 8) & 0xFF;

            }

            return decoded_data;

        };

How to crop a CvMat in OpenCV?

To get better results and robustness against differents types of matrices, you can do this in addition to the first answer, that copy the data :

cv::Mat source = getYourSource();

// Setup a rectangle to define your region of interest
cv::Rect myROI(10, 10, 100, 100);

// Crop the full image to that image contained by the rectangle myROI
// Note that this doesn't copy the data
cv::Mat croppedRef(source, myROI);

cv::Mat cropped;
// Copy the data into new matrix
croppedRef.copyTo(cropped);

Returning http 200 OK with error within response body

I think these kinds of problems are solved if we think about real life.

Bad Practice:

Example 1:

Darling everything is FINE/OK (HTTP CODE 200) - (Success):
{
  ...but I don't want us to be together anymore!!!... (Error)
  // Then everything isn't OK???
}

Example 2:

You are the best employee (HTTP CODE 200) - (Success):
{
  ...But we cannot continue your contract!!!... (Error)
  // Then everything isn't OK???
}

Good Practices:

 Darling I don't feel good (HTTP CODE 400) - (Error):
{
  ...I no longer feel anything for you, I think the best thing is to separate... (Error)
  // In this case, you are alerting me from the beginning that something is wrong ...
}

This is only my personal opinion, each one can implement it as it is most comfortable or needs.

Note: The idea for this explanation was drawn from a great friend @diosney

How to use if-else logic in Java 8 stream forEach

Just put the condition into the lambda itself, e.g.

animalMap.entrySet().stream()
        .forEach(
                pair -> {
                    if (pair.getValue() != null) {
                        myMap.put(pair.getKey(), pair.getValue());
                    } else {
                        myList.add(pair.getKey());
                    }
                }
        );

Of course, this assumes that both collections (myMap and myList) are declared and initialized prior to the above piece of code.


Update: using Map.forEach makes the code shorter, plus more efficient and readable, as Jorn Vernee kindly suggested:

    animalMap.forEach(
            (key, value) -> {
                if (value != null) {
                    myMap.put(key, value);
                } else {
                    myList.add(key);
                }
            }
    );

How do I get logs/details of ansible-playbook module executions?

The playbook script task will generate stdout just like the non-playbook command, it just needs to be saved to a variable using register. Once we've got that, the debug module can print to the playbook output stream.

tasks:
- name: Hello yourself
  script: test.sh
  register: hello

- name: Debug hello
  debug: var=hello

- name: Debug hello.stdout as part of a string
  debug: "msg=The script's stdout was `{{ hello.stdout }}`."

Output should look something like this:

TASK: [Hello yourself] ******************************************************** 
changed: [MyTestHost]

TASK: [Debug hello] *********************************************************** 
ok: [MyTestHost] => {
    "hello": {
        "changed": true, 
        "invocation": {
            "module_args": "test.sh", 
            "module_name": "script"
        }, 
        "rc": 0, 
        "stderr": "", 
        "stdout": "Hello World\r\n", 
        "stdout_lines": [
            "Hello World"
        ]
    }
}

TASK: [Debug hello.stdout as part of a string] ******************************** 
ok: [MyTestHost] => {
    "msg": "The script's stdout was `Hello World\r\n`."
}

Find first and last day for previous calendar month in SQL Server Reporting Services (VB.Net)

in C#:

new DateTime(DateTime.Now.Year, DateTime.Now.Month, 1).AddMonths(-1)
new DateTime(DateTime.Now.Year, DateTime.Now.Month, 1).AddDays(-1)

iOS 11, 12, and 13 installed certificates not trusted automatically (self signed)

Recommended solution is to install and trust a self-signed certificate (root). Assuming you created your own CA and the hierarchy of the certificated is correct you don't need to change the server trust evaluation. This is recommended because it doesn't require any changes in the code.

  1. Generate CA and the certificates (you can use openssl: Generating CA and self-signed certificates.
  2. Install root certificate (*.cer file) on the device - you can open it by Safari and it should redirect you to Settings
  3. When the certificated is installed, go to Certificate Trust Settings (Settings > General > About > Certificate Trust Settings) as in MattP answer.

If it is not possible then you need to change server trust evaluation.

More info in this document: Technical Q&A QA1948 HTTPS and Test Servers

UICollectionView - dynamic cell height?

TL;DR: Scan down to image, and then check out working project here.

Updating my answer for a simpler solution that I found..

In my case, I wanted to fix the width, and have variable height cells. I wanted a drop in, reusable solution that handled rotation and didn't require a lot of intervention.

What I arrived at, was override (just) systemLayoutFitting(...) in the collection cell (in this case a base class for me), and first defeat UICollectionView's effort to set the wrong dimension on contentView by adding a constraint for the known dimension, in this case, the width.

class EstimatedWidthCell: UICollectionViewCell {
    override init(frame: CGRect) {
        super.init(frame: frame)
        contentView.translatesAutoresizingMaskIntoConstraints = false
    }

    required init?(coder aDecoder: NSCoder) {
        super.init(coder: aDecoder)
        contentView.translatesAutoresizingMaskIntoConstraints = false
    }

    override func systemLayoutSizeFitting(
        _ targetSize: CGSize, withHorizontalFittingPriority
        horizontalFittingPriority: UILayoutPriority,
        verticalFittingPriority: UILayoutPriority) -> CGSize {

        width.constant = targetSize.width

and then return the final size for the cell - used for (and this feels like a bug) the dimension of the cell itself, but not contentView - which is otherwise constrained to a conflicting size (hence the constraint above). To calculate the correct cell size, I use a lower priority for the dimension that I wanted to float, and I get back the height required to fit the content within the width to which I want to fix:

        let size = contentView.systemLayoutSizeFitting(
            CGSize(width: targetSize.width, height: 1),
            withHorizontalFittingPriority: .required,
            verticalFittingPriority: verticalFittingPriority)

        print("\(#function) \(#line) \(targetSize) -> \(size)")
        return size
    }

    lazy var width: NSLayoutConstraint = {
        return contentView.widthAnchor
            .constraint(equalToConstant: bounds.size.width)
            .isActive(true)
    }()
}

But where does this width come from? It is configured via the estimatedItemSize on the collection view's flow layout:

lazy var collectionView: UICollectionView = {
    let view = UICollectionView(frame: CGRect(), collectionViewLayout: layout)
    view.backgroundColor = .cyan
    view.translatesAutoresizingMaskIntoConstraints = false
    return view
}()

lazy var layout: UICollectionViewFlowLayout = {
    let layout = UICollectionViewFlowLayout()
    let width = view.bounds.size.width // should adjust for inset
    layout.estimatedItemSize = CGSize(width: width, height: 10)
    layout.scrollDirection = .vertical
    return layout
}()

Finally, to handle rotation, I implement trailCollectionDidChange to invalidate the layout:

override func traitCollectionDidChange(_ previousTraitCollection: UITraitCollection?) {
    layout.estimatedItemSize = CGSize(width: view.bounds.size.width, height: 10)
    layout.invalidateLayout()
    super.traitCollectionDidChange(previousTraitCollection)
}

The final result looks like this:

enter image description here

And I have published a working sample here.

How can I catch a ctrl-c event?

You have to catch the SIGINT signal (we are talking POSIX right?)

See @Gab Royer´s answer for sigaction.

Example:

#include <signal.h>
#include <stdlib.h>
#include <stdio.h>

void my_handler(sig_t s){
           printf("Caught signal %d\n",s);
           exit(1); 

}

int main(int argc,char** argv)
{
   signal (SIGINT,my_handler);

   while(1);
   return 0;

}

Object spread vs. Object.assign

This isn't necessarily exhaustive.

Spread syntax

options = {...optionsDefault, ...options};

Advantages:

  • If authoring code for execution in environments without native support, you may be able to just compile this syntax (as opposed to using a polyfill). (With Babel, for example.)

  • Less verbose.

Disadvantages:

  • When this answer was originally written, this was a proposal, not standardized. When using proposals consider what you'd do if you write code with it now and it doesn't get standardized or changes as it moves toward standardization. This has since been standardized in ES2018.

  • Literal, not dynamic.


Object.assign()

options = Object.assign({}, optionsDefault, options);

Advantages:

  • Standardized.

  • Dynamic. Example:

    var sources = [{a: "A"}, {b: "B"}, {c: "C"}];
    options = Object.assign.apply(Object, [{}].concat(sources));
    // or
    options = Object.assign({}, ...sources);
    

Disadvantages:

  • More verbose.
  • If authoring code for execution in environments without native support you need to polyfill.

This is the commit that made me wonder.

That's not directly related to what you're asking. That code wasn't using Object.assign(), it was using user code (object-assign) that does the same thing. They appear to be compiling that code with Babel (and bundling it with Webpack), which is what I was talking about: the syntax you can just compile. They apparently preferred that to having to include object-assign as a dependency that would go into their build.

Can't draw Histogram, 'x' must be numeric

Because of the thousand separator, the data will have been read as 'non-numeric'. So you need to convert it:

 we <- gsub(",", "", we)   # remove comma
 we <- as.numeric(we)      # turn into numbers

and now you can do

 hist(we)

and other numeric operations.

Prevent flex items from overflowing a container

max-width works for me.

aside {
  flex: 0 1 200px;
  max-width: 200px;
}

Variables of CSS pre-processors allows to avoid hard-coding.

aside {
  $WIDTH: 200px;
  flex: 0 1 $WIDTH;
  max-width: $WIDTH;
}

overflow: hidden also works, but I lately I try do not use it because it hides the elements as popups and dropdowns.

While loop in batch

@echo off

set countfiles=10

:loop

set /a countfiles -= 1

echo hi

if %countfiles% GTR 0 goto loop

pause

on the first "set countfiles" the 10 you see is the amount it will loop the echo hi is the thing you want to loop

...i'm 5 years late

XSLT counting elements with a given value

This XPath:

count(//Property[long = '11007'])

returns the same value as:

count(//Property/long[text() = '11007'])

...except that the first counts Property nodes that match the criterion and the second counts long child nodes that match the criterion.

As per your comment and reading your question a couple of times, I believe that you want to find uniqueness based on a combination of criteria. Therefore, in actuality, I think you are actually checking multiple conditions. The following would work as well:

count(//Property[@Name = 'Alive'][long = '11007'])

because it means the same thing as:

count(//Property[@Name = 'Alive' and long = '11007'])

Of course, you would substitute the values for parameters in your template. The above code only illustrates the point.

EDIT (after question edit)


You were quite right about the XML being horrible. In fact, this is a downright CodingHorror candidate! I had to keep recounting to keep track of the "Property" node I was on presently. I feel your pain!

Here you go:

count(/root/ac/Properties/Property[Properties/Property/Properties/Property/long = $parPropId])

Note that I have removed all the other checks (for ID and Value). They appear not to be required since you are able to arrive at the relevant node using the hierarchy in the XML. Also, you already mentioned that the check for uniqueness is based only on the contents of the long element.

Set cursor position on contentEditable <div>

Update

I've written a cross-browser range and selection library called Rangy that incorporates an improved version of the code I posted below. You can use the selection save and restore module for this particular question, although I'd be tempted to use something like @Nico Burns's answer if you're not doing anything else with selections in your project and don't need the bulk of a library.

Previous answer

You can use IERange (http://code.google.com/p/ierange/) to convert IE's TextRange into something like a DOM Range and use it in conjunction with something like eyelidlessness's starting point. Personally I would only use the algorithms from IERange that do the Range <-> TextRange conversions rather than use the whole thing. And IE's selection object doesn't have the focusNode and anchorNode properties but you should be able to just use the Range/TextRange obtained from the selection instead.

I might put something together to do this, will post back here if and when I do.

EDIT:

I've created a demo of a script that does this. It works in everything I've tried it in so far except for a bug in Opera 9, which I haven't had time to look into yet. Browsers it works in are IE 5.5, 6 and 7, Chrome 2, Firefox 2, 3 and 3.5, and Safari 4, all on Windows.

http://www.timdown.co.uk/code/selections/

Note that selections may be made backwards in browsers so that the focus node is at the start of the selection and hitting the right or left cursor key will move the caret to a position relative to the start of the selection. I don't think it is possible to replicate this when restoring a selection, so the focus node is always at the end of the selection.

I will write this up fully at some point soon.

How to update UI from another thread running in another class

Everything that interacts with the UI must be called in the UI thread (unless it is a frozen object). To do that, you can use the dispatcher.

var disp = /* Get the UI dispatcher, each WPF object has a dispatcher which you can query*/
disp.BeginInvoke(DispatcherPriority.Normal,
        (Action)(() => /*Do your UI Stuff here*/));

I use BeginInvoke here, usually a backgroundworker doesn't need to wait that the UI updates. If you want to wait, you can use Invoke. But you should be careful not to call BeginInvoke to fast to often, this can get really nasty.

By the way, The BackgroundWorker class helps with this kind of taks. It allows Reporting changes, like a percentage and dispatches this automatically from the Background thread into the ui thread. For the most thread <> update ui tasks the BackgroundWorker is a great tool.

How can I create an observable with a delay

What you want is a timer:

// RxJS v6+
import { timer } from 'rxjs';

//emit [1, 2, 3] after 1 second.
const source = timer(1000).map(([1, 2, 3]);
//output: [1, 2, 3]
const subscribe = source.subscribe(val => console.log(val));

Where is NuGet.Config file located in Visual Studio project?

There are multiple nuget packages read in the following order:

  1. First the NuGetDefaults.Config file. You will find this in %ProgramFiles(x86)%\NuGet\Config.
  2. The computer-level file.
  3. The user-level file. You will find this in %APPDATA%\NuGet\nuget.config.
  4. Any file named nuget.config beginning from the root of your drive up to the directory where nuget.exe is called.
  5. The config file you specify in the -configfile option when calling nuget.exe

You can find more information here.

Copy files without overwrite

For %F In ("C:\From\*.*") Do If Not Exist "C:\To\%~nxF" Copy "%F" "C:\To\%~nxF"

input file appears to be a text format dump. Please use psql

If you have a full DB dump:

PGPASSWORD="your_pass" psql -h "your_host" -U "your_user" -d "your_database" -f backup.sql

If you have schemas kept separately, however, that won't work. Then you'll need to disable triggers for data insertion, akin to pg_restore --disable-triggers. You can then use this:

cat database_data_only.gzip | gunzip | PGPASSWORD="your_pass" psql -h "your_host" -U root "your_database" -c 'SET session_replication_role = replica;' -f /dev/stdin

On a side note, it is a very unfortunate downside of postgres, I think. The default way of creating a dump in pg_dump is incompatible with pg_restore. With some additional keys, however, it is. WTF?

Excel VBA Run Time Error '424' object required

The first code line, Option Explicit means (in simple terms) that all of your variables have to be explicitly declared by Dim statements. They can be any type, including object, integer, string, or even a variant.

This line: Dim envFrmwrkPath As Range is declaring the variable envFrmwrkPath of type Range. This means that you can only set it to a range.

This line: Set envFrmwrkPath = ActiveSheet.Range("D6").Value is attempting to set the Range type variable to a specific Value that is in cell D6. This could be a integer or a string for example (depends on what you have in that cell) but it's not a range.

I'm assuming you want the value stored in a variable. Try something like this:

Dim MyVariableName As Integer
MyVariableName = ActiveSheet.Range("D6").Value

This assumes you have a number (like 5) in cell D6. Now your variable will have the value.

For simplicity sake of learning, you can remove or comment out the Option Explicit line and VBA will try to determine the type of variables at run time.


Try this to get through this part of your code

Dim envFrmwrkPath As String
Dim ApplicationName As String
Dim TestIterationName As String

error C2039: 'string' : is not a member of 'std', header file problem

Take care not to include

#include <string.h> 

but only

#include <string>

It took me 1 hour to find this in my code.

Hope this can help

Excel error HRESULT: 0x800A03EC while trying to get range with cell's name

I ran to a similar error running Excel in VBA, what I've learned is that when I pull data from MSSQL, and then using get_range and .Value2 apply it's out of the range, any value that was of type uniqueidentifier (GUID) resulted in this error. Only when I cast the value to nvarcahr(max) it worked.

Cannot open include file: 'stdio.h' - Visual Studio Community 2017 - C++ Error

Faced the problem of missing stdlib.h and stdio.h (and maybe more) after installing VS2017 Community on a new computer and migrating a solution from VS2013 to VS2017.

Used @Maxim Akristiniy's proposal, but still got error message regarding toolset compatibility. However VS itself suggested to do solution retarget by right-clicking on the solution in Solution Explorer, then selecting Retarget solution from the menu and the updated Windows SDK Version from the drop-down list.

Now my projects build w/o a problem.

Note that you may need to make the project your startup project for the retargeting to catch.

String length in bytes in JavaScript

Years passed and nowadays you can do it natively

(new TextEncoder().encode('foo')).length

Note that it's not supported by IE (you may use a polyfill for that).

MDN documentation

Standard specifications

AngularJS sorting rows by table header

I'm just getting my feet wet with angular, but I found this great tutorial.
Here's a working plunk I put together with credit to Scott Allen and the above tutorial. Click search to display the sortable table.

For each column header you need to make it clickable - ng-click on a link will work. This will set the sortName of the column to sort.

<th>
     <a href="#" ng-click="sortName='name'; sortReverse = !sortReverse">
          <span ng-show="sortName == 'name' && sortReverse" class="glyphicon glyphicon-triangle-bottom"></span>
          <span ng-show="sortName == 'name' && !sortReverse" class="glyphicon glyphicon-triangle-top"></span>
           Name
     </a>
</th>

Then, in the table body you can pipe in that sortName in the orderBy filter orderBy:sortName:sortReverse

<tr ng-repeat="repo in repos | orderBy:sortName:sortReverse | filter:searchRepos">
     <td>{{repo.name}}</td>
     <td class="tag tag-primary">{{repo.stargazers_count | number}}</td>
     <td>{{repo.language}}</td>
</tr>

ReferenceError: variable is not defined

Got the error (in the function init) with the following code ;

"use strict" ;

var hdr ;

function init(){ // called on load
    hdr = document.getElementById("hdr");
}

... while using the stock browser on a Samsung galaxy Fame ( crap phone which makes it a good tester ) - userAgent ; Mozilla/5.0 (Linux; U; Android 4.1.2; en-gb; GT-S6810P Build/JZO54K) AppleWebKit/534.30 (KHTML, like Gecko) Version/4.0 Mobile Safari/534.30

The same code works everywhere else I tried including the stock browser on an older HTC phone - userAgent ; Mozilla/5.0 (Linux; U; Android 2.3.5; en-gb; HTC_WildfireS_A510e Build/GRJ90) AppleWebKit/533.1 (KHTML, like Gecko) Version/4.0 Mobile Safari/533.1

The fix for this was to change

var hdr ;

to

var hdr = null ;

Multiple maven repositories in one gradle file

In short you have to do like this

repositories {
  maven { url "http://maven.springframework.org/release" }
  maven { url "https://maven.fabric.io/public" }
}

Detail:

You need to specify each maven URL in its own curly braces. Here is what I got working with skeleton dependencies for the web services project I’m going to build up:

apply plugin: 'java'

sourceCompatibility = 1.7
version = '1.0'

repositories {
  maven { url "http://maven.springframework.org/release" }
  maven { url "http://maven.restlet.org" }
  mavenCentral()
}

dependencies {
  compile group:'org.restlet.jee', name:'org.restlet', version:'2.1.1'
  compile group:'org.restlet.jee', name:'org.restlet.ext.servlet',version.1.1'
  compile group:'org.springframework', name:'spring-web', version:'3.2.1.RELEASE'
  compile group:'org.slf4j', name:'slf4j-api', version:'1.7.2'
  compile group:'ch.qos.logback', name:'logback-core', version:'1.0.9'
  testCompile group:'junit', name:'junit', version:'4.11'
}

Blog

How to filter a RecyclerView with a SearchView

Following @Shruthi Kamoji in a cleaner way, we can just use a filterable, its meant for that:

public abstract class GenericRecycleAdapter<E> extends RecyclerView.Adapter implements Filterable
{
    protected List<E> list;
    protected List<E> originalList;
    protected Context context;

    public GenericRecycleAdapter(Context context,
    List<E> list)
    {
        this.originalList = list;
        this.list = list;
        this.context = context;
    }

    ...

    @Override
    public Filter getFilter() {
        return new Filter() {
            @SuppressWarnings("unchecked")
            @Override
            protected void publishResults(CharSequence constraint, FilterResults results) {
                list = (List<E>) results.values;
                notifyDataSetChanged();
            }

            @Override
            protected FilterResults performFiltering(CharSequence constraint) {
                List<E> filteredResults = null;
                if (constraint.length() == 0) {
                    filteredResults = originalList;
                } else {
                    filteredResults = getFilteredResults(constraint.toString().toLowerCase());
                }

                FilterResults results = new FilterResults();
                results.values = filteredResults;

                return results;
            }
        };
    }

    protected List<E> getFilteredResults(String constraint) {
        List<E> results = new ArrayList<>();

        for (E item : originalList) {
            if (item.getName().toLowerCase().contains(constraint)) {
                results.add(item);
            }
        }
        return results;
    }
} 

The E here is a Generic Type, you can extend it using your class:

public class customerAdapter extends GenericRecycleAdapter<CustomerModel>

Or just change the E to the type you want (<CustomerModel> for example)

Then from searchView (the widget you can put on menu.xml):

searchView.setOnQueryTextListener(new SearchView.OnQueryTextListener() {
    @Override
    public boolean onQueryTextSubmit(String text) {
        return false;
    }

    @Override
    public boolean onQueryTextChange(String text) {
        yourAdapter.getFilter().filter(text);
        return true;
    }
});

change type of input field with jQuery

Just create a new field to bypass this security thing:

var $oldPassword = $("#password");
var $newPassword = $("<input type='text' />")
                          .val($oldPassword.val())
                          .appendTo($oldPassword.parent());
$oldPassword.remove();
$newPassword.attr('id','password');

CSS/HTML: Create a glowing border around an Input Field

input[type="text"]{
   @include transition(all 0.30s ease-in-out);
  outline: none;
  padding: 3px 0px 3px 3px;
  margin: 5px 1px 3px 0px;
  border: 1px solid #DDDDDD;
}
input[type="text"]:focus{
  @include box-shadow(0 0 5px rgba(81, 203, 238, 1));
  -webkit-box-shadow: 0px 0px 5px #007eff;  
  -moz-box-shadow: 0px 0px 5px #007eff;  
  box-shadow: 0px 0px 5px #007eff;
}

Adding a collaborator to my free GitHub account?

project link:

https://github.com/your_username/you_repo_name/settings

you will get a page like this, go to Collaborator and add collaborator Check setting tab

android layout with visibility GONE

<LinearLayout
    android:layout_width="match_parent"
    android:layout_height="wrap_content"
    android:layout_below="@+id/activity_register_header"
    android:minHeight="50dp"
    android:orientation="vertical"
    android:visibility="gone" />

Try this piece of code..For me this code worked..

SVN Error - Not a working copy

Workaround: Rename directory which is not 'working copy' Checkout/update/restore this directory again Move files from renamed directory to new Commit changes

Reason: You made some changes to some files under .svn directory, this breaks 'working copy'

What is the best way to remove the first element from an array?

An alternative ugly method:

   String[] a ={"BLAH00001","DIK-11","DIK-2","MAN5"};
   String[] k=Arrays.toString(a).split(", ",2)[1].split("]")[0].split(", ");

Adding subscribers to a list using Mailchimp's API v3

If you Want to run Batch Subscribe on a List using Mailchimp API . Then you can use the below function.

    /**
     * Mailchimp API- List Batch Subscribe added function
     *
     * @param array  $data   Passed you data as an array format.
     * @param string $apikey your mailchimp api key.
     *
     * @return mixed
     */
    function batchSubscribe(array $data, $apikey)
    {
        $auth          = base64_encode('user:' . $apikey);
        $json_postData = json_encode($data);
        $ch            = curl_init();
        $dataCenter    = substr($apikey, strpos($apikey, '-') + 1);
        $curlopt_url   = 'https://' . $dataCenter . '.api.mailchimp.com/3.0/batches/';
        curl_setopt($ch, CURLOPT_URL, $curlopt_url);
        curl_setopt($ch, CURLOPT_HTTPHEADER, array('Content-Type: application/json',
            'Authorization: Basic ' . $auth));
        curl_setopt($ch, CURLOPT_USERAGENT, 'PHP-MCAPI/3.0');
        curl_setopt($ch, CURLOPT_RETURNTRANSFER, true);
        curl_setopt($ch, CURLOPT_TIMEOUT, 10);
        curl_setopt($ch, CURLOPT_POST, true);
        curl_setopt($ch, CURLOPT_SSL_VERIFYPEER, false);
        curl_setopt($ch, CURLOPT_POSTFIELDS, $json_postData);
        $result = curl_exec($ch);
        return $result;
    }

Function Use And Data format for Batch Operations:

<?php
$apikey  = 'Your MailChimp Api Key';
$list_id = 'Your list ID';
$servername = 'localhost';
$username   = 'Youre DB username';
$password   = 'Your DB password';
$dbname     = 'Your DB Name';
// Create connection
$conn = new mysqli($servername, $username, $password, $dbname);
// Check connection
if ($conn->connect_error) {
    die('Connection failed: ' . $conn->connect_error);
}
$sql       = 'SELECT * FROM emails';// your SQL Query goes here
$result    = $conn->query($sql);
$finalData = [];
if ($result->num_rows > 0) {
    // output data of each row
    while ($row = $result->fetch_assoc()) {
        $individulData = array(
            'apikey'        => $apikey,
            'email_address' => $row['email'],
            'status'        => 'subscribed',
            'merge_fields'  => array(
                'FNAME' => 'eastwest',
                'LNAME' => 'rehab',
            )
        );
        $json_individulData        = json_encode($individulData);
        $finalData['operations'][] =
            array(
                "method" => "POST",
                "path"   => "/lists/$list_id/members/",
                "body"   => $json_individulData
            );
    }
}
$api_response = batchSubscribe($finalData, $apikey);
print_r($api_response);
$conn->close();

Also, You can found this code in my Github gist. GithubGist Link

Reference Documentation: Official

Windows Batch: How to add Host-Entries?

I am adding this answer in case someone else would like to store the host entry set in a txt file formatted like the normal host file. This looks for a TAB delimiter. This is based off of the answers from @Rashy and @that0n3guy. The differences can be noticed around the FOR command.

@echo off
TITLE Modifying your HOSTS file
ECHO.

:: BatchGotAdmin
:-------------------------------------
REM  --> Check for permissions
>nul 2>&1 "%SYSTEMROOT%\system32\cacls.exe" "%SYSTEMROOT%\system32\config\system"

REM --> If error flag set, we do not have admin.
if '%errorlevel%' NEQ '0' (
    echo Requesting administrative privileges...
    goto UACPrompt
) else ( goto gotAdmin )

:UACPrompt
    echo Set UAC = CreateObject^("Shell.Application"^) > "%temp%\getadmin.vbs"
    set params = %*:"="
    echo UAC.ShellExecute "cmd.exe", "/c %~s0 %params%", "", "runas", 1 >> "%temp%\getadmin.vbs"

    "%temp%\getadmin.vbs"
    del "%temp%\getadmin.vbs"
    exit /B

:gotAdmin
    pushd "%CD%"
    CD /D "%~dp0"
:--------------------------------------

:LOOP
SET Choice=
SET /P Choice="Do you want to modify HOSTS file ? (Y/N)"

IF NOT '%Choice%'=='' SET Choice=%Choice:~0,1%

ECHO.
IF /I '%Choice%'=='Y' GOTO ACCEPTED
IF /I '%Choice%'=='N' GOTO REJECTED
ECHO Please type Y (for Yes) or N (for No) to proceed!
ECHO.
GOTO Loop


:REJECTED
ECHO Your HOSTS file was left unchanged.
ECHO Finished.
GOTO END


:ACCEPTED
setlocal enabledelayedexpansion
::Create your list of host domains
for /F "tokens=1,2 delims=  " %%A in (%WINDIR%\System32\drivers\etc\storedhosts.txt) do (
    SET _host=%%B
    SET _ip=%%A
    SET NEWLINE=^& echo.
    ECHO Adding !_ip!       !_host!
    REM REM ::strip out this specific line and store in tmp file
    type %WINDIR%\System32\drivers\etc\hosts | findstr /v !_host! > tmp.txt
    REM REM ::re-add the line to it
    ECHO %NEWLINE%^!_ip!        !_host! >> tmp.txt
    REM ::overwrite host file
    copy /b/v/y tmp.txt %WINDIR%\System32\drivers\etc\hosts
    del tmp.txt 
)

ipconfig /flushdns
ECHO.
ECHO.
ECHO Finished, you may close this window now.
GOTO END

:END
ECHO.
PAUSE
EXIT

Example "storedhosts.txt" (tab delimited)

127.0.0.1   mysite.com
168.1.64.2  yoursite.com
192.1.0.1   internalsite.com

Removing an item from a select box

You can delete the selected item with this:

$("#selectBox option:selected").remove();

This is useful if you have a list and not a dropdown.

Append key/value pair to hash with << in Ruby

Similar as they are, merge! and store treat existing hashes differently depending on keynames, and will therefore affect your preference. Other than that from a syntax standpoint, merge!'s key: "value" syntax closely matches up against JavaScript and Python. I've always hated comma-separating key-value pairs, personally.

hash = {}
hash.merge!(key: "value")
hash.merge!(:key => "value")
puts hash

{:key=>"value"}

hash = {}
hash.store(:key, "value")
hash.store("key", "value")
puts hash

{:key=>"value", "key"=>"value"}

To get the shovel operator << working, I would advise using Mark Thomas's answer.

Dynamic instantiation from string name of a class in dynamically imported module?

Use getattr to get an attribute from a name in a string. In other words, get the instance as

instance = getattr(modul, class_name)()

Can I calculate z-score with R?

if x is a vector with raw scores then scale(x) is a vector with standardized scores.

Or manually: (x-mean(x))/sd(x)

Is there are way to make a child DIV's width wider than the parent DIV using CSS?

I tried some of your solutions. This one :

margin: 0px -100%;
padding: 0 100%;

is by far the best, since we don't need extra css for smaller screen. I made a codePen to show the results : I used a parent div with a background image, and a child divon div with inner content.

https://codepen.io/yuyazz/pen/BaoqBBq

Converting between datetime, Timestamp and datetime64

indeed, all of these datetime types can be difficult, and potentially problematic (must keep careful track of timezone information). here's what i have done, though i admit that i am concerned that at least part of it is "not by design". also, this can be made a bit more compact as needed. starting with a numpy.datetime64 dt_a:

dt_a

numpy.datetime64('2015-04-24T23:11:26.270000-0700')

dt_a1 = dt_a.tolist() # yields a datetime object in UTC, but without tzinfo

dt_a1

datetime.datetime(2015, 4, 25, 6, 11, 26, 270000)

# now, make your "aware" datetime:

dt_a2=datetime.datetime(*list(dt_a1.timetuple()[:6]) + [dt_a1.microsecond], tzinfo=pytz.timezone('UTC'))

... and of course, that can be compressed into one line as needed.

Is there a way to get colored text in GitHubflavored Markdown?

You cannot include style directives in GFM.

The most complete documentation/example is "Markdown Cheatsheet", and it illustrates that this element <style> is missing.

If you manage to include your text in one of the GFM elements, then you can play with a github.css stylesheet in order to colors that way, meaning to color using inline CSS style directives, referring to said css stylesheet.

Angular ng-repeat Error "Duplicates in a repeater are not allowed."

The solution is actually described here: http://www.anujgakhar.com/2013/06/15/duplicates-in-a-repeater-are-not-allowed-in-angularjs/

AngularJS does not allow duplicates in a ng-repeat directive. This means if you are trying to do the following, you will get an error.

// This code throws the error "Duplicates in a repeater are not allowed.
// Repeater: row in [1,1,1] key: number:1"
<div ng-repeat="row in [1,1,1]">

However, changing the above code slightly to define an index to determine uniqueness as below will get it working again.

// This will work
<div ng-repeat="row in [1,1,1] track by $index">

Official docs are here: https://docs.angularjs.org/error/ngRepeat/dupes

Converting a pointer into an integer

Use uintptr_t as your integer type.

Excel Macro - Select all cells with data and format as table

Try this one for current selection:

Sub A_SelectAllMakeTable2()
    Dim tbl As ListObject
    Set tbl = ActiveSheet.ListObjects.Add(xlSrcRange, Selection, , xlYes)
    tbl.TableStyle = "TableStyleMedium15"
End Sub

or equivalent of your macro (for Ctrl+Shift+End range selection):

Sub A_SelectAllMakeTable()
    Dim tbl As ListObject
    Dim rng As Range

    Set rng = Range(Range("A1"), Range("A1").SpecialCells(xlLastCell))
    Set tbl = ActiveSheet.ListObjects.Add(xlSrcRange, rng, , xlYes)
    tbl.TableStyle = "TableStyleMedium15"
End Sub

Swift: Display HTML data in a label or textView

Here is a Swift 3 version:

private func getHtmlLabel(text: String) -> UILabel {
    let label = UILabel()
    label.numberOfLines = 0
    label.lineBreakMode = .byWordWrapping
    label.attributedString = stringFromHtml(string: text)
    return label
}

private func stringFromHtml(string: String) -> NSAttributedString? {
    do {
        let data = string.data(using: String.Encoding.utf8, allowLossyConversion: true)
        if let d = data {
            let str = try NSAttributedString(data: d,
                                             options: [NSDocumentTypeDocumentAttribute: NSHTMLTextDocumentType],
                                             documentAttributes: nil)
            return str
        }
    } catch {
    }
    return nil
}

I found issues with some of the other answers here and it took me a bit to get this right. I set the line break mode and number of lines so that the label sized appropriately when the HTML spanned multiple lines.

Adjust width of input field to its input

Here is my 2 cents. Create an empty invisible div. Fill it with the input content and return the width to the input field. Match text styles between each box.

_x000D_
_x000D_
$(".answers_number").keyup(function(){_x000D_
    $( "#number_box" ).html( $( this ).val() );_x000D_
    $( this ).animate({_x000D_
        width: $( "#number_box" ).width()+20_x000D_
        }, 300, function() {_x000D_
    });_x000D_
});
_x000D_
#number_box {_x000D_
   position: absolute;_x000D_
   visibility: hidden;_x000D_
   height: auto;_x000D_
   width: auto;_x000D_
   white-space: nowrap;_x000D_
   padding:0 4px;_x000D_
   /*Your font styles to match input*/_x000D_
   font-family:Arial;_x000D_
   font-size: 30px; _x000D_
}_x000D_
    _x000D_
.answers_number {_x000D_
   font-size: 30px; _x000D_
   font-family:Arial;_x000D_
}
_x000D_
<script src="https://ajax.googleapis.com/ajax/libs/jquery/2.1.1/jquery.min.js"></script>_x000D_
<input type="number" class="answers_number" />_x000D_
<div id="number_box">_x000D_
</div>
_x000D_
_x000D_
_x000D_

CSS fixed width in a span

<style type="text/css">

span {
  position:absolute;
  width: 50px;
}

</style>

You can do this method for assigning width for inline elements

git index.lock File exists when I try to commit, but cannot delete the file

On Linux, Unix, Git Bash, or Cygwin, try:

rm -f .git/index.lock

On Windows Command Prompt, try:

del .git\index.lock


For Windows:

  • From a PowerShell console opened as administrator, try

    rm -Force ./.git/index.lock
    
  • If that does not work, you must kill all git.exe processes

    taskkill /F /IM git.exe
    

    SUCCESS: The process "git.exe" with PID 20448 has been terminated.
    SUCCESS: The process "git.exe" with PID 11312 has been terminated.
    SUCCESS: The process "git.exe" with PID 23868 has been terminated.
    SUCCESS: The process "git.exe" with PID 27496 has been terminated.
    SUCCESS: The process "git.exe" with PID 33480 has been terminated.
    SUCCESS: The process "git.exe" with PID 28036 has been terminated. \

    rm -Force ./.git/index.lock
    

Change border-bottom color using jquery?

to modify more css property values, you may use css object. such as:

hilight_css = {"border-bottom-color":"red", 
               "background-color":"#000"};
$(".msg").css(hilight_css);

but if the modification code is bloated. you should consider the approach March suggested. do it this way:

first, in your css file:

.hilight { border-bottom-color:red; background-color:#000; }
.msg { /* something to make it notifiable */ }

second, in your js code:

$(".msg").addClass("hilight");
// to bring message block to normal
$(".hilight").removeClass("hilight");

if ie 6 is not an issue, you can chain these classes to have more specific selectors.

How to hide Table Row Overflow?

In general, if you are using white-space: nowrap; it is probably because you know which columns are going to contain content which wraps (or stretches the cell). For those columns, I generally wrap the cell's contents in a span with a specific class attribute and apply a specific width.

Example:

HTML:

<td><span class="description">My really long description</span></td>

CSS:

span.description {
    display: inline-block;
    overflow: hidden;
    white-space: nowrap;
    width: 150px;
}

Background color for Tk in Python

config is another option:

widget1.config(bg='black')
widget2.config(bg='#000000')

or:

widget1.config(background='black')
widget2.config(background='#000000')

Replacing spaces with underscores in JavaScript?

try this:

key=key.replace(/ /g,"_");

that'll do a global find/replace

javascript replace

In Javascript, how do I check if an array has duplicate values?

Well I did a bit of searching around the internet for you and I found this handy link.

Easiest way to find duplicate values in a JavaScript array

You can adapt the sample code that is provided in the above link, courtesy of "swilliams" to your solution.

Can you use CSS to mirror/flip text?

Just one more example how the character could be flipped. Add vendor prefixes if you need ones but for now all modern browsers support unprefixed transform property. The only exception is Opera if Opera Mini mode is enabled (~3% world users).

_x000D_
_x000D_
<!DOCTYPE html>_x000D_
<html>_x000D_
<head>_x000D_
  <meta charset="utf-8">_x000D_
  <title>Text rotation</title>_x000D_
  <style type="text/css" media="screen">_x000D_
    .scissors {_x000D_
      display: inline-block;_x000D_
      font-size: 50px;_x000D_
      color: red;_x000D_
    }_x000D_
    .original {_x000D_
      color: initial;_x000D_
    }_x000D_
    .flipped {_x000D_
      transform: rotateZ(180deg);_x000D_
    }_x000D_
    .upward {_x000D_
      transform: rotateZ(-90deg);_x000D_
    }_x000D_
    .downward {_x000D_
      transform: rotateZ(90deg);_x000D_
    }_x000D_
  </style>_x000D_
  _x000D_
</head>_x000D_
<body>_x000D_
  <ul>_x000D_
    <li>Original: <span class="scissors original">&#9986;</span></li>_x000D_
    <li>Flipped: <span class="scissors flipped">&#9986;</span></li>_x000D_
    <li>Upward: <span class="scissors upward">&#9986;</span></li>_x000D_
    <li>Downward: <span class="scissors downward">&#9986;</span></li>_x000D_
  </ul>_x000D_
</body>_x000D_
</html>
_x000D_
_x000D_
_x000D_

How to run vi on docker container?

Use below command in Debian based container:

apt-get install vim-tiny

Complete instruction for using in Dockerfile:

RUN apt-get update && apt-get install --no-install-recommends -y \   
 vim-tiny \  
 && apt-get clean && rm -rf /var/lib/apt/lists/*

It doesn't install unnecessary packages and removes unnecessary downloaded files, so your docker image size won't increase dramatically.

find index of an int in a list

It's even easier if you consider that the Generic List in C# is indexed from 0 like an array. This means you can just use something like:

int index = 0; int i = accounts[index];

JQuery: Change value of hidden input field

Seems to work

$(".selector").change(function() {

    var $value = $(this).val();

    var $title = $(this).children('option[value='+$value+']').html();

    $('#bacon').val($title);

});

Just check with your firebug. And don't put css on hidden input.

Limit length of characters in a regular expression?

(^(\d{2})|^(\d{4})|^(\d{5}))$

This expression takes the number of length 2,4 and 5. Valid Inputs are 12 1234 12345

Set type for function parameters?

It can easilly be done with ArgueJS:

function myFunction ()
{
  arguments = __({myDate: Date, myString: String});
  // do stuff
};

How to convert BigInteger to String in java

Why don't you use the BigInteger(String) constructor ? That way, round-tripping via toString() should work fine.

(note also that your conversion to bytes doesn't explicitly specify a character-encoding and is platform-dependent - that could be source of grief further down the line)

How to generate sample XML documents from their DTD or XSD?

XMLBlueprint 7.5 can do the following: - generate sample xml from dtd - generate sample xml from relax ng schema - generate sample xml from xml schema

C++ compile error: has initializer but incomplete type

` Please include either of these:

`#include<sstream>`

using std::istringstream; 

android - How to get view from context?

Why don't you just use a singleton?

import android.content.Context;


public class ClassicSingleton {
    private Context c=null;
    private static ClassicSingleton instance = null;
    protected ClassicSingleton()
    {
       // Exists only to defeat instantiation.
    }
    public void setContext(Context ctx)
    {
    c=ctx;
    }
    public Context getContext()
    {
       return c;
    }
    public static ClassicSingleton getInstance()
    {
        if(instance == null) {
            instance = new ClassicSingleton();
        }
        return instance;
    }
}

Then in the activity class:

 private ClassicSingleton cs = ClassicSingleton.getInstance();

And in the non activity class:

ClassicSingleton cs= ClassicSingleton.getInstance();
        Context c=cs.getContext();
        ImageView imageView = (ImageView) ((Activity)c).findViewById(R.id.imageView1);

How can I get (query string) parameters from the URL in Next.js?

Using Next.js 9 or above you can get query parameters:

With router:

import { useRouter } from 'next/router'

const Index = () => {
  const router = useRouter()
  const {id} = router.query

  return(<div>{id}</div>)
}

With getInitialProps:

const Index = ({id}) => {
  return(<div>{id}</div>)
}

Index.getInitialProps = async ({ query }) => {
  const {id} = query

  return {id}
}

Firing events on CSS class changes in jQuery

Whenever you change a class in your script, you could use a trigger to raise your own event.

$(this).addClass('someClass');
$(mySelector).trigger('cssClassChanged')
....
$(otherSelector).bind('cssClassChanged', data, function(){ do stuff });

but otherwise, no, there's no baked-in way to fire an event when a class changes. change() only fires after focus leaves an input whose input has been altered.

_x000D_
_x000D_
$(function() {_x000D_
  var button = $('.clickme')_x000D_
      , box = $('.box')_x000D_
  ;_x000D_
  _x000D_
  button.on('click', function() { _x000D_
    box.removeClass('box');_x000D_
    $(document).trigger('buttonClick');_x000D_
  });_x000D_
            _x000D_
  $(document).on('buttonClick', function() {_x000D_
    box.text('Clicked!');_x000D_
  });_x000D_
});
_x000D_
.box { background-color: red; }
_x000D_
<script src="https://ajax.googleapis.com/ajax/libs/jquery/2.1.1/jquery.min.js"></script>_x000D_
_x000D_
<div class="box">Hi</div>_x000D_
<button class="clickme">Click me</button>
_x000D_
_x000D_
_x000D_

More info on jQuery Triggers

Reading in double values with scanf in c

Format specifier in printf should be %f for doubl datatypes since float datatyles eventually convert to double datatypes inside printf.

There is no provision to print float data. Please find the discussion here : Correct format specifier for double in printf